Está en la página 1de 450

Lactante de 10 meses de edad, es llevado al centro de salud por


presencia de prurito en cara. A la exploración se encuentran las
siguientes lesiones. El diagnóstico más probable en este caso es:

A dermatitis por contacto

B eccema atópico

C dermatitis seborréica

D eccema numular

Paciente de 62 años de edad, el cual acude a consulta por presentar
las siguientes lesiones en piel. La lesión dermatológica que presenta el
paciente deberá considerarse:

A un exantema fototóxico

B una eritrodermia

C un eritema multiforme

D un pénfigo vulgar

Hombre de 35 años de edad, acude al servicio de urgencias por la
presencia de dolor agudo tipo cólico en región lumbar derecha que se
irradia hacia la fosa ilíaca y escroto del mismo lado. El examen de
orina resulta negativo para infección, unicamente con hemoglobina
positiva. La radiografía abdominal muestra la siguiente imagen. Es
muy probable que la imagen radiopaca de la radiografía esté
relacionada con un lito compuesto por:

A urato sódico

B fosfato de calcio

C fosfato amino magnesio

D xantinas

Hombre de 58 años de edad, jubilado en condición de abandono.
Refiere disnea de esfuerzo de al menos un año de evolución, la cual
ha sido progresiva y que se ha hecho acompañar en los últimos 6
meses de tos seca. Combe negativo. A la exploración física se
auscultan estertores inspiratorios tardíos difusos, se observan dedos
en palillo de tambor. Se solicita radiografía de tórax encontrando la
siguiente imagen. Con base en los antecedentes y los hallazgos
radiológicos se integra el diagnóstico de:

A fibrosis pulmonar idiopática

B neumonía intersticial aguda

C esclerosis generalizada progresiva

D proteinosis alveolar pulmonar

La lesión pulmonar es secundaria a:

A acumulación de material lipoproteináceo amorfo alveolar

B reacción autoinmunitaria mediada por igg

C inflamación crónica con aparición de cicatrices de fibrosis

D sustitución progresiva de la estructura hística normal por tejido
conjuntivo denso

Hombre de 48 años de edad, desde hace 3 años le fue diagnosticada
diabetes mellitus, negándose a recibir tratamiento farmacológico. Es
llevado por sus familiares a consulta tras presentar aumento de
volumen en hemicara derecha y halitosis severa. A la exploración de
la cavidad oral se observa la siguiente imagen. El diagnóstico más
probable es:

A candidiasis invasiva

B mucormicosis rinocerebral

C otitis externa maligna

D sinusitis crónica

El agente causal involucrado en este caso es:

A pseudomona aeruginosa

B rhizopus oryzae

C cándida albicans

D moraxella catarrhalis

Mujer de 46 años de edad, con diagnóstico de infección por el VIH,
acude a consulta por presencia de las siguientes lesiones en la piel.
Las lesiones dérmicas que presenta la paciente son características de
la siguiente enfermedad:

A eritema discoide

B lepra tuberculoide

C eccema numular

D eritema multiforme

Mujer de 22 años de edad, nulípara, con antecedente de dolor pélvico
crónico. Se realiza ultrasonido pélvico, el cual reporta una masa de
apariencia quística a nivel de ovario izquierdo mayor a 5 cm. Se realiza
cirugía laparoscópica para resección de la tumoración, la cual se
rompe a la manipulación, encontrando los siguientes hallazgos. Con
base en el antecedente y el hallazgo quirúrgico, el diagnóstico más
probable es:

A tumor mucinoso

B quiste tecaluteínico

C endometrioma

Hombre de 35 años de edad, el cual acude a consulta por presentar
tos productiva de varios meses de evolución, en ocasiones
acompañada de rasgos de sangre. La tomografía de pulmón muestra
la siguiente imagen. Con base en los antecedentes y los hallazgos
tomográficos es posible integrar el diagnóstico de:

A aspergilosis pulmonar

B tuberculosis cavitaria

C cáncer broncoalveolar

D criptococosis

Deberá indicarse el siguiente tratamiento de elección:

A anfotericina b

B antifímicos

C quimiorradiación

D voriconazol

Hombre de 56 años de edad, diabético descontrolado. Refiere úlcera
en pie de al menos 4 meses de evolución con empeoramiento
progresivo, a su llegada con absceso y abundante secreción purulenta,
la radiografía no muestra imágenes sugestivas de ostomielitis. Tras
una curación simple, la lesión se observa de la siguiente manera. Con
base en la clasificación de wagner el paciente cursa con un pie
diabético:

A grado 2

B grado 3

C grado 4

D grado 5

En este caso de no responder al manejo conservador, el tratamiento a
seguir deberá ser mediante:

A revascularización

B desbridamiento quirúrgico

C amputación parcial

D amputación supracondilea

Durante su rotación por el servicio de endoscopia, se le realiza un
estudio a paciente masculino de 56 años de edad, encontrando a
nivel del esófago el siguiente hallazgo. Con base en el estudio
endoscópico el diagnóstico más probable es:

A esófago de barrett

B acalasia

C várices esofágicas

D pólipo esofágico

Hombre de 48 años de edad, diabético descontrolado, hace unas
semanas presentó infección limitada por herpes zoster, la cual se
resolvió favorablemente solo con administración de aciclovir por 5
días. Actualmente acude a consulta por presencia de lesiones
ampollosas y dolorosas en tronco, espalda y boca. La piel de estas
lesiones se desprende fácilmente. A la exploración se encuentran los
siguientes hallazgos. El diagnóstico más probable es:

A pénfigo vulgar

B impétigo

C necrosis epidérmica tóxica

D síndrome de steven johnson

Hombre de 28 años de edad, acude a la consulta refiriendo prurito en
el pene. A la exploración genital se observan las siguientes lesiones. El
diagnóstico más probable es:

A fimosis

B balanitis

C liquen plano

D enfermedad de bowen

Deberá indicarse el siguiente tratamiento:

A glucocorticoides tópicos

B azitromicina

C nistatina tópica

D 5- fluorouracilo

Preescolar de 4 años de edad. Actualmente en estudio por masa
abdominal palpable, hematuria y fiebre. Se realiza tomografía
abdominal encontrando el siguiente hallazgo. Con base en la
sintomatología y la imagen tomográfica, el diagnóstico más probable
es:

A tumor renal de células claras

B nefroblastoma

C mesoblastoma congénito

D tumor rabdoide maligno
Dado que no es un estadio avanzado, deberá indicarse el siguiente
tratamiento de inicio:

A quimioterapia preoperatoria

B resección de órganos en bloque

C radioterapia

D nefrectomía radical

Hombre de 23 años de edad, el cual sufre caída de al menos 3 metros
de altura mientras realizaba piruetas en una competencia de
acrobacias en bicicleta. Actualmente consciente, policontundido, se
queja de dolor e incapacidad para la movilización completa de los
dedos de la mano izquierda. Las placas radiográficas muestran la
siguiente imagen: El siguiente es el mecanismo más probable de
lesión:

A tracción

B hiperextensión

C rotación

D flexión

Mujer de aproximadamente 25 años de edad, es encontrada
inconsciente en la calle con golpes contusos en todo el cuerpo. A su
ingreso al servicio de urgencias se toman radiografías de tórax y
abdomen encontrando los siguientes hallazgos: Con base en las
imágenes radiográficas la paciente presenta:

A hemoperitoneo

B íleo paralítico agudo

C peritonitis bacteriana espontánea

D neumoperitoneo

Hombre de 65 años de edad, el cual es llevado por vecinos al servicio
de urgencias tras sufrir caída de su propia altura en la entrada de su
domicilio. A la exploración física se observa caquéctico, somnoliento,
con palidez de piel, presenta tos expectorante y hemoptisis en una
ocasión, se auscultan estertores difusos en ambos pulmones,
abdomen blando sin organomegalias. Se solicita radiografía de tórax
encontrando el siguiente hallazgo radiológico. Con base en la
exploración física y la imagen radiológica, el diagnóstico probable del
paciente es el de:


A tuberculosis miliar

B metástasis pulmonares

C neumopatía intersticial

El principal antecedente que debe ser investigado de forma
intencionada en el paciente es

A la presencia de tumor sólido primario

B la convivencia con personas con tuberculosis activa

C la exposición ocupacional a agentes nocivos pulmonares

D el tabaquismo crónico

Paciente masculino de 38 años de edad, llega al servicio de urgencias
por quemadura en las extremidades inferiores. Lo más probable es
que las lesiones son secundarias a una quemadura:

A química

B por radiación

C solar

D por corriente eléctrica

Escolar de 8 años de edad, es llevado a consulta por presencia de
dolor faríngeo y tos. A la exploración se observa la siguiente imagen.
Lo más probable es que los hallazgos encontrados estén asociados a
una infección por:

A estreptococo b-hemolítico del grupo a

B virus del epstein barr

C corynebacterium diphtheriae

D virus de la influenza

El tratamiento de elección para este caso consiste en la
administración de:

A oseltamivir

B penicilina v

C antitoxina específica

D paracetamol

Mujer de 48 años de edad. Acude a consulta refiriendo dolor en ojo
derecho, que se incrementa con el movimiento de los párpados
acompañandose de fotofobia y visión borrosa. A la exploración física
se observa la siguiente imagen. El siguiente es el diagnóstico más
probable:

A conjuntivitis

B uveítis

C catarata

D queratitis

Mujer de 27 años de edad, acude para toma de citología cervical
anual. Refiere desde hace una semana dolor pélvico acompañado de
leucorrea. A la exploración vaginal se observa la siguiente imagen. El
agente etiológico más probable es:

A gardnerella vaginalis

B trichomona vaginalis

C chlamydia trachomatis

D cándida albicans

El tratamiento más indicado es con:

A miconazol

B azitromicina

C clindamicina

D metronidazol

Hombre de 25 años de edad, el cual desde hace 5 años es piloto de
una avioneta para aspersión de pesticidas. Acude a consulta
refiriendo epistaxis frecuente y aparición de hematomas sin causa
aparente. Los estudios de sangre periférica revelan anemia
importante con eritrocitos anormalmente grandes, sin reticulocitos y
plaquetopenia. Como parte del protocolo se realiza biopsia de médula
ósea observando la siguiente imagen. Con base en los antecedentes y
los datos aportados por la biopsia de médula ósea el diagnóstico
probable es el de:

A púrpura trombocitopénica inmunológica

B anemia aplásica

C anemia megaloblástica

Mujer de 57 años de edad, con antecedente de síndrome metabólico
con mal apego al tratamiento. Es llevada al servicio de urgencias tras
presentar dolor torácico sugestivo de isquemia cardiaca. Durante su
ingreso se corrobora el diagnóstico de infarto mediante marcadores
bioquímicos y electrocardiograma. Con base en el trazo
electrocardiográfico podemos deducir que la ubicación del infarto es:

A anterior

B lateral

C posterior

D inferior

Es importante recordar que la unidad estructural y funcional de la
contracción cardiaca es:

A el miocito

B la sarcómera

C la miosina

D el miocardio

Mujer de 38 años de edad, acude al servicio de urgencias por la
presencia de dolor en flanco derecho que se irradia a la región
inguinal del mismo lado y vómito gastroalimentario en una ocasión.
Se solicita una radiografía abdominal encontrando la siguiente
imagen. El diagnóstico más probable es:

A colecistitis aguda

B urolitiasis

C apendicitis aguda

D oclusión intestinal

Con base en el diagnóstico deberá indicarse en este momento:


A analgesia

B apendicectomía

C colecistectomía laparoscópica

D sonda nasogástrica

Adolescente obesa de 14 años de edad. Acude a consulta por dolor de
inicio súbito de 6 horas de evolución que inicia en hipogastrio y
después se localiza en la fosa ilíaca derecha, se acompaña de náuseas
y malestar general. A la exploración física con temperatura 38.5°c,
abdomen distendido, Mc burney positivo. Se solicita radiografía
simple de abdomen. El diagnóstico más probable deberá apoyarse en
el hallazgo del siguiente signo radiológico presente en la imagen:

A borramiento del psoas

B asa centinela

C coprostasis difusa

D grano de café

Mujer de 43 años de edad, cuenta con antecedente de colecistitis
crónica. Acude al servicio de urgencias por presentar dolor en
cuadrante superior derecho, náuseas y vómito. A la exploración física
se encuentra febril, con signo de murphy positivo y resistencia
muscular en cuadrante superior derecho. La radiografía simple de
abdomen muestra los siguientes hallazgos: La radiografía de abdomen
muestra una imagen compatible con:

A íleo biliar

B colecistitis litiásica

C colecistitis enfisematosa

D vesícula de porcelana

Hombre de 35 años de edad, el cual ingresa al servicio de urgencias
tras accidente automovilístico hace 30 minutos. Clínicamente con
datos de dificultad respiratoria. La radiografía de tórax muestra la
siguiente imagen. Con base en los antecedentes y los hallazgos
radiológicos deberá sospecharse la presencia de:

A neumotórax

B neumopericardio

C hemotórax

D contusión pulmonar

Mujer de 27 años de edad, la cual acude a consulta muy preocupada
por la presencia de descamación de la piel cabelluda. Refiere haber
utilizado remedios caseros sin mejoría clínica. A la exploración física
se encuentra la siguiente lesión: Con base en las lesiones dermicas, el
diagnóstico más probable es:

A tiña capitis

B psoriasis

C dermatitis seborreica

D candidosis

El tratamiento que debe iniciarse en esta paciente es con:


A miconazol tópico

B hidrocortisona + flutrimazol tópico

C terbinafina tópica

D hidrocortisona + ácido salicílico tópicos

Hombre de 32 años de edad, el cual ingresa al servicio de urgencias
tras haber recibido un impacto de bala por arma de fuego en el
abdomen, se sospecha la presencia de perforación de víscera hueca.
La radiografía abdominal inicial muestra la siguiente imagen: El signo
radiológico que se señala en esta imagen se denomina:

A signo de balón de rugby

B signo de rigler

C signo del triángulo

D signo de la cúpula

Hombre de 28 años de edad, con antecedente de síndrome de
inmunodeficiencia adquirida, sin alcance de metas terapéuticas en los
últimos 6 meses. Acude a urgencias por presentar dolor precordial
progresivo y disnea. A la exploración física: febril, con facies dolorosa,
taquipnea y taquicardia. El electrocardiograma muestra elevación
difusa de st, depresión difusa de pr e inversión de las ondas t. Se
solicita ecocardiograma. El hallazgo ecográfico encontrado es
compatible con:

A engrosamiento pericárdico

B derrame pericárdico

C neumopericardio

D tumor cardiaco

Hombre de 32 años de edad, con antecedente de criptorquidia
derecha, la cual fue corregida antes del año de edad. Refiere dolor
sordo en testículo derecho desde hace 1 mes. Se le indica ultrasonido
testicular encontrando el siguiente hallazgo. El diagnóstico más
probable es:

A orquiepididimitis

B torsión de apéndice testicular

C tumor maligno testicular

D hidrocele comunicante

Hombre de 32 años de edad quien sufre quemadura eléctrica por
contacto con cables de alta tensión mientras se encontraba realizando
trabajos en la azotea de una vivienda. A su llegada al hospital se toma
un trazo electrocardiográfico que muestra la siguiente imagen. Con
base en los antecedentes y los hallazgos electrocardiográficos el
paciente presenta:

A ritmo idioventricular acelerado

B taquicardia sinusal inapropiada

C fibrilación ventricular

D taquicardia ventricular polimorfa

El lugar donde debe generarse de forma normal el impulso eléctrico
cardiaco es en:

A nodo sinoauricular

B nodo aurículoventricular

C en las aurículas

D en el sistema his-purkinje.

Mujer de 38 años de edad, vih positivo, sin antecedentes
neurológicos. Es llevada a consulta por presentar cefalea y
alteraciones agudas en la visión. Se realiza tomografía computada
encontrando la siguiente imagen. Con base en la sintomatología y la
imagen encontrada en la tomografía, el diagnóstico más probable es:

A encefalitis toxoplásmica

B enfermedad hidatídica

C criptococosis meníngea

D neurocisticercosis

El agente infeccioso más probablemente involucrado en este caso es:

A toxoplasma gondii

B cryptococcus neoformans

C echinococcus granulosus

D taenia solium

Hombre de 27 años de edad. Refiere presencia de algunas lesiones
dermatológicas pruriginosas, en tórax y abdomen desde hace al
menos una semana. Al rascarse la piel del paciente, aparece la
siguiente reacción. A esta reacción de la piel se le conoce con el
nombre de:

A signo de darier

B signo de nikolsky

C fenómeno de koebner

D signo de gottron

Mujer de 23 años de edad. Es enviada al servicio de dermatología por
presentar lesiones máculo papulares confluentes y pruriginosas en la
piel expuesta al sol, aún cuando la exposición fue leve. Al momento
de la consulta, la paciente presenta únicamente las siguientes lesiones
faciales. Lo más probable es que en este momento la paciente
presenta:

A una dermatitis lúpica

B una quemadura solar

C una rosácea


El siguiente es el tratamiento de elección:

A ácido retinóico oral

B hidroxicloroquina tópica

C glucocorticoides orales

D metrotexate

Hombre de 65 años de edad, el cual trabajó durante al menos 30 años
en una marmolería cortando piedra. Acude a consulta refiriendo tos y
expectoración crónica. La radiografía de tórax muestra la siguiente
imagen. El diagnóstico pulmonar más probable es:

A silicosis

B enfisema pulmonar

C tuberculosis miliar

D criptococosis

La lesión pulmonar presente es secundaria a:

A destrucción alveolar sin presencia de fibrosis

B reacción inflamatoria a polisacáridos capsulares

C diseminación hematológica de la micobacteria al tejido pulmonar

D sustitución del tejido pulmonar por colágeno

Mujer de 23 años de edad, acude a solicitar atención médica por la
presencia de disnea progresiva. A la exploración con desplazamiento
del ápex mas allá de la línea medio clavicular y estertores crepitantes
bilaterales. La radiografía de tórax muestra la siguiente imagen. Lo
más probable es que se trate de una cardiopatía:

A chagásica

B reumática

C hipertensiva

D pulmonar

El factor de riesgo de mayor importancia que debemos buscar de
forma intencionada en este caso es:

A hipertensión arterial crónica

B enfermedad pulmonar obstructiva crónica

C infecciones faríngeas recurrentes

D vivir en zona endémica para chagas

Mujer de 21 años de edad, con datos de desnutrición severa. Acude
en este momento a consulta por presentar dolor abdominal
postprandial desde hace 48 hrs que mejora con el vómito. Cuenta con
el antecedente de haber egresado hace cuatro meses del hospital,
con diagnóstico de síndrome de gran quemado. Desde su egreso
hasta la fecha ha presentado pérdida de peso de al menos 10 kg. Se
realiza un estudio radiológico contrastado encontrando los siguientes
hallazgos. Con base en los antecedentes y los hallazgos radiológicos el
diagnóstico más probable es:

A carcinoma de la ampolla de vater

B adenocarcinoma duodenal

C adenoma de las glándulas de brunner

D síndrome de la arteria mesentérica superior

Niña de 3 años 6 meses de edad, hija de madre adicta a la cocaína. Es
llevada a consulta por notar desde hace un mes proptosis del ojo
derecho. Durante la exploración física se encuentra el siguiente
hallazgo clínico. Por su elevada frecuencia en este grupo de edad, el
diagnóstico más probable es el de un:

A hemangioma

B rabdomiosarcoma

C linfangioma

D neurofibroma

Hombre de 33 años de edad, con infección por VIH desde hace 3
años. Es llevado a urgencias por presentar convulsiones generalizadas.
La resonancia magnética muestra la siguiente imagen. Las imágenes
que presenta el paciente en la RM son compatibles con la siguiente
patología:

A criptococosis

B isquemia cerebral múltiple

C cisticercosis

D toxoplasmosis

El tratamiento de elección en este caso es:


A trimetoprim- sulfametoxazol

B anfotericina b

C trombolisis

D praziquantel

Recién nacida femenina, con antecedente de prematurez y
polihidramnios. Al cumplir las primeras 12 hrs de vida presenta
vómito de contenido gastrobiliar. Se realiza una radiografía simple de
abdomen, encontrando el siguiente hallazgo. El siguiente es el
diagnóstico más probable:

A atresia yeyunoileal

B íleo meconial

C duplicación intestinal

D atresia duodenal

Hombre de 45 años de edad, con disminución de al menos dos tallas
en 3 meses. Acude a consulta refiriendo tos con expectoración escasa
de más de un mes, en la última semana se agrega sensación de
hipertermia nocturna. La radiografía de tórax muestra la siguiente
imagen. El diagnóstico más probable es:

A tuberculosis cavitaria

B metástasis pulmonares

C criptococosis

D cáncer broncoalveolar

Deberá indicarse el siguiente tratamiento:

A quimioterapia específica para tumor primario

B antifímicos

C antimicotico

D quimiorradiación

Mujer de 71 años, previamente sana, que ayer tuvo fractura de
cadera tras una caída. Se encontraba esperando tiempo quirúrgico
cuando desarrolló disnea súbita, hipotensión y desaturación de
oxígeno hasta 72% (SO2 basal de 91%). ¿Cuál es el hallazgo
electrocardiográfico más probable en este caso?

A Taquicardia sinusal

B S1Q3T3

C Bloqueo de rama derecha

D Taquicardia helicoidal

Hombre de 30 años de edad que acude a consulta por palpitaciones
regulares de 5 minutos durante el esfuerzo moderado. Un tío materno
murió de muerte súbita a los 25 años de edad. Refiere fatiga de
grandes esfuerzos desde hace 1 año. A la exploración tiene frecuencia
cardiaca de 60 lpm, presión arterial de 130/80, pulsos carotideos
bífidos, soplo sistólico en borde esternal izquierdo que se irradia hacia
el borde esternal izquierdo bajo, que aumenta de intensidad con la
maniobra de Valsalva y disminuye con el empuñamiento de las
manos. El EKG muestra ritmo sinusal, datos de hipertrofia ventricular
izquierda y ondas q en derivaciones inferiores. ¿Cuál es el diagnóstico
más probable en este paciente?

A Esclerosis aórtica

B Miocardiopatía hipertrófica

C Estenosis aórtica congénita

Un niño de 6 años de edad es llevado a consulta por disnea de
esfuerzo. Tiene antecedentes de infección de vías respiratorias bajas
en forma frecuente y los padres refieren fatiga de moderados
esfuerzos desde hace 2 años. FC: 90 lpm, presión arterial normal, bajo
peso para la edad, levantamiento paraesternal izquierdo bajo, ápex
desplazado hacia abajo y soplo sistólico “en barra” en el borde
paraesternal izquierdo que se irradia a toda al área precordial. En el
EKG se observa ritmo sinusal y datos de hipertrofia ventricular
izquierda y derecha. ¿Cuál es el diagnóstico más probable en este
paciente?

A Comunicación interventricular

B Foramen oval permeable

C Persistencia del conducto arterioso

D Comunicación interauricular

Mujer de 67 años con antecedente de insuficiencia cardiaca izquierda
de 3 años de evolución. Ingresa al servicio de urgencias porque hace
una hora inició con dificultad respiratoria, diaforesis, palidez
generalizada, tos con expectoración color salmón. TA 150/90, FC 114
lpm, FR 35, temp 35.8°C, ingurgitación yugular, campos pulmonares
con estertores crepitantes infraescapulares bilaterales. ¿Cuál es el
diagnóstico más probable?

A Edema agudo de pulmón

B Insuficiencia cardiaca derecha descompensada

C Insuficiencia renal aguda

D Tromboembolia pulmonar

Un hombre de 57 años acude a consulta por disnea de esfuerzo. Tiene
antecedente de diabetes mellitus tipo 2 desde hace 10 años. Se le
hace diagnóstico de hipertensión arterial y actualmente tiene
170/100mmHg. ¿En qué estadio de la clasificación de la hipertensión
arterial de acuerdo al JNC7 está el paciente?

A A

B B

C 1*

D 2*

En relación al caso anterior, ¿Cuál es la forma más recomendable de
iniciar el tratamiento?

A Un antihipertensivo a dosis máxima

B Tres antihipertensivos

C Sólo medidas generales

D Dos antihipertensivos

En relación al caso anterior, durante el estudio inicial se hace el
diagnóstico de Nefropatía diabética. ¿Cuál es la cifra meta
recomendada con el tratamiento antihipertensivo?


A <100/70

B <140/90

C <145/85

D <130/80

En relación al mismo caso, ¿Cuál fármaco antihipertensivo es de
elección?


A Enalapril

B Labetalol

C Clonidina

D Hidralazina

Un hombre de 60 años acude a urgencias por dolor precordial. Tiene
antecedentes de infarto del miocardio y cirugía de revascularización
coronaria hace 5 años, hipertensión arterial, diabetes mellitus tipo 2 e
insuficiencia renal crónica. Toma aspirina, metoprolol y enalapril.
Refiere dos episodios de dolor precordial durante horas de la
madrugada en reposo de 15 minutos de duración y se presenta con
dolor tipo anginoso de 1h de duración. A la exploración tiene FC de 86
lpm, presión arterial de 160/90 y cuarto ruido cardiaco constante.
Este es su electrocardiograma: ¿Cuál es el siguiente paso en el
abordaje de este paciente?

A Angiografía coronaria

B Estudio de perfusión miocárdica

C Determinación de mioglobina y CPK

D Determinación de troponinas y CPK-MB

Hombre de 65 años de edad que acude a consulta por dolor
precordial al esfuerzo físico moderado. Tiene antecedentes de
tabaquismo e hipertensión arterial crónica. Refiere disnea de
moderados esfuerzos desde hace 6 meses y 2 episodios previos de
síncope durante el esfuerzo. A la exploración tiene frecuencia cardiaca
de 60 lpm, presión arterial de 150/90, pulsos carotideos y periféricos
disminuidos en amplitud, soplo telesistólico en foco aórtico de
moderada intensidad, que se irradia a vasos del cuello y cuarto ruido
cardiaco constante. ¿Cuál es el diagnóstico más probable?

A Insuficiencia mitral

B Estenosis mitral

C Estenosis aórtica

D Estenosis de arteria pulmonar

Una niña de 6 años de edad es llevada a consulta por disnea. Tiene
antecedentes de haber nacido prematuramente. Hace 3 años
comenzó con disnea de esfuerzos, la cual ha progresado de los
grandes a medianos esfuerzos. A la exploración tiene frecuencia
cardiaca de 86 lpm, presión arterial de 130/60 mm Hg, levantamiento
paraesternal izquierdo prominente, soplo continuo (en maquinaria) y
de tono grave en el segundo espacio intercostal izquierdo. El ECG
demuestra ritmo sinusal y datos de hipertrofia ventricular izquierda.
¿Cuál es el diagnóstico más probable en esta paciente?

A Comunicación interauricular

B Coartación de la aorta

C Persistencia de conducto arterioso

D Comunicación interventricular

Una mujer de 58 años acude a urgencias por dolor precordial durante
el esfuerzo físico. Padece diabetes, hipertensión e hiperlipidemia
mixta. Refiere disnea de grandes esfuerzos desde hace un año,
acompañada en ocasiones de dolor precordial de tipo opresivo, de
moderada intensidad, irradiado hacia el cuello, de 5 minutos de
duración, que cede con el reposo. FC: 76 lpm, TA: 150/90 mm Hg y
cuarto ruido cardiaco constante. El EKG muestra ritmo sinusal y datos
de hipertrofia del ventrículo izquierdo. ¿Cuál es el estudio de elección
para el diagnóstico en esta paciente?

A Electrocardiograma de esfuerzo en banda

B Estudio de perfusión miocárdica con radionúclido

C Ecocardiograma transtorácico en reposo

D Angiotomografía de coronarias

Un hombre de 76 años de edad se presenta a urgencias por dolor
precordial durante el esfuerzo. Tiene antecedentes de tabaquismo,
hipertensión arterial y obesidad. Refiere dolor tipo opresivo en región
precordial desde hace 2 meses, asociado a esfuerzos físicos
moderados, irradiado al brazo izquierdo, de 5 minutos de duración y
que cede con el reposo. Hace 1 semana ha notado que el dolor se
presenta durante esfuerzos leves, es de 30 minutos de duración. A la
exploración tiene frecuencia cardiaca de 64 lpm, presión arterial de
130/80, ruidos cardiacos con cuarto ruido cardiaco constante y soplo
sistólico en foco mitral de ligera intensidad. El ECG muestra ritmo
sinusal y no se observan anormalidades. ¿Cuál es el diagnóstico
clínico de este paciente?

A Angina inestable

B Angina estable crónica

C Angina de esfuerzo estable

El monitor muestra asistolia, pero el interno refiere le había tomado
este electrocardiograma dos horas antes. Ante estos datos ¿Qué
consideraría como causa de la asistolia?

A Infarto agudo de miocardio

B Cetoacidosis diabética

C Hipokalemia

D Hiperkalemia

Hombre de 65 años de edad que acude a urgencias por dolor
precordial. Tiene antecedentes de tabaquismo e hipertensión arterial.
Refiere angina de moderados esfuerzos desde hace 6 meses. A su
ingreso refiere dolor precordial opresivo desde hace 1 hora, intenso,
irradiado a cuello y acompañado de disnea leve y nausea. A la
exploración tiene frecuencia cardiaca de 90 lpm, presión arterial de
145/90, diaforesis y palidez generalizada, ruidos cardiacos con cuarto
ruido cardiaco contante y estertores subcrepitantes discretos en
ambas bases pulmonares. Este es su EKG ¿Cuál es el diagnóstico de
este paciente?

A Disección aórtica

B Infarto agudo de miocardio

C Tromboembolia pulmonar

D Pericarditis aguda

Mujer de 45 años de edad que acude a consulta por fatiga de esfuerzo
y edema de miembros inferiores. Tiene antecedente de 2 episodios
de tromboembolia pulmonar y desde entonces toma anticoagulante
oral. Refiere disnea de moderados esfuerzos desde hace 8 meses y
edema de miembros inferiores desde hace 2 meses. FC:82 lpm, TA:
100/60 mm Hg, plétora yugular grado II/IV, soplo sistólico en foco
tricuspídeo, reforzamiento pulmonar del segundo ruido cardiaco,
sibilancias en bases pulmonares, hepatomegalia, reflujo
hepatoyugular y edema de extremidades inferiores. El EKG en ritmo
sinusal muestra datos dilatación de ventrículo y aurícula derechos
¿Qué signos a la exploración sugieren el diagnóstico de insuficiencia
cardiaca derecha?

A Reforzamiento pulmonar del segundo ruido

B Plétora yugular y hepatomegalia

C Presión arterial diastólica <60 mm Hg

Hombre de 46 años a quién usted está por iniciarle a ziprazidona (un
antipsicótico atípico). ¿Qué estudio es necesario realizarle a un
paciente al que se le inicia dicho medicamento?

A Electroencefalograma

B Ultrasonografía de vías biliares

C Electrocardiograma

D Química sanguínea

Hombre de 52 años, con antecedente de EPOC por tabaquismo de
más de 30 años. Presenta desde hace 7 días palpitaciones y disnea. A
la exploración presenta FC 139/min, TA 90/60 mmHg, FR: 32/min. El
electrocardiograma reporta ondas p con tres morfologías diferentes
en la misma derivación, ritmo irregular. ¿Qué representa la alteración
electrocardiográfica?

A Disfunción del nodo aurículo-ventricular.

B Disfunción del nodo sinusal.

C Presencia de focos ectópicos de activación eléctrica.


Hombre de 85 años con EVC isquémico de 2 hrs de evolución. Como
parte de su abordaje se le tomó el siguiente electrocardiograma.
¿Cuál de los siguientes diagnósticos es el más probable?

A Fibrilación auricular

B Flutter auricular

C Taquicardia ventricular monomorfa

D Taquicardia nodal

Hombre de 69 años que padece insuficiencia cardiaca, quien para su
tratamiento recibe múltiples fármacos. Este es su electrocardiograma,
¿cuál es el diagnóstico más probable?

A Bloqueo incompleto de rama derecha

B Bloqueo AV de 1er grado

C Bradicardia sinusal

D Bloqueo AV de segundo grado

Se obtiene el siguiente electrocardiograma a los 7 minutos de su
ingreso al servicio de urgencias. ¿Cuál es el mejor siguiente paso?

A Enviar a un hospital con unidad de cuidados coronarios

B Iniciar fibrinolisis

C Iniciar inhibidores de Glucoproteína IIb/IIIa

D Esperar resultados de troponinas

Mujer de 59 años, padece diabetes tipo 2, hiperlipidemia y tuvo un
IAM de la circunfleja derecha el año pasado. Acude a consulta por
lipotimia. En la tira del monitor se encontró lo siguiente. ¿Cuál es el
diagnóstico más probable?

A Bloqueo AV de primer grado

B Bloqueo AV de tercer grado

C Bloqueo AV de segundo grado tipo 2

D Bloqueo AV de segundo grado tipo 1

Hombre de 64 años, que padece hipertensión arterial. Acude consulta
de revisión con su médico, quien revisa el siguiente
electrocardiograma. ¿Cuál de las siguientes opciones es compatible
con este electrocardiograma?

A Bloqueo completo de rama izquierda del haz de His

B Hipertrofia ventricular derecha

C Bloqueo completo de rama derecha del haz de His

D Hipertrofia ventricular izquierda

Un hombre de 69 años de edad con antecedentes de hipertensión
arterial sistémica, en tratamiento con metoprolol de liberación
prolongada desde hace 3 años, se presenta a consulta externa por
astenia y cefalea. A la exploración tiene frecuencia cardiaca de 48lpm,
presión arterial de 110/86. El Electrocardiograma muestra lo
siguiente: ¿Qué grado de bloqueo auriculoventricular tiene este
paciente?

A Bloqueo auriculoventricular de tercer grado

B Bloqueo auriculoventricular de segundo grado tipo II

C Bloqueo auriculoventricular de segundo grado tipo I

D Bloqueo auriculoventricular completo

Hombre de 46 años que tuvo un síncope mientras jugaba fútbol con
sus sobrinos. No se refiere enfermo pero su vida es sedentaria,
padece obesidad y tiene antecedentes familiares de infarto agudo de
miocardio de su padre a los 60 años. Con el resultado de este
electrocardiograma, ¿cuál es el diagnóstico más probable?

A Bloqueo AV de tercer grado

B Síndrome del seno enfermo

C Bloqueo AV de segundo grado tipo 2

D Bloqueo de rama derecha incompleto

Mujer de 42 años, se encuentra en valoración de cirugía bariátrica
para tratar la obesidad mórbida. Se le realizó el siguiente
electrocardiograma, ¿Cuál es la anomalía detectada?

A Bloqueo completo de rama izquierda del haz de His

B Bloqueo de fascículo posterior de rama izquierda del haz de His

C Bloqueo de fascículo anterior de rama izquierda del haz de His

D Bloqueo completo de rama derecha del Haz de His

Las manifestaciones clínicas de la intoxicación por organofosforados
en escolares son secundarias a:

A Inhibición de la acetilcolinesterasa por fosforilación

B Aumento de la actividad de los receptores muscarinicos

C Aumento de la actividad de los receptores nicotinicos

D Aumento de la actividad de la esterasa neurotóxica

El antibiótico de elección para el tratamiento de la neumonía atípica
es la:

A Amoxicilina

B Tetraciclina

C Amikacina

D Claritromicina

La difenilhidantoina ejerce su acción antiepiléptica debido a que
produce:

A Aumento de la velocidad de recuperación de los canales de sodio

B Aumento de la activación repetitiva de los potenciales de acción

C Disminución de las reacciones al ácido-gamma-amino-butírico

D Retraso de la velocidad de recuperación de los canales de sodio

En la intoxicación por ácido acetilsalicílico se presenta:

A Parálisis muscular progresiva que afecta los músculos respiratorios

B Hiperventilación, disminución de CO2 y liberación de ácido láctico
tisular

C Hipotermia, rigidez muscular y temblor en reposo

D Aumento de la adhesividad plaquetaria con riesgo de
tromboembolia

Un hombre de 45 años previamente sano, inicia su padecimiento un
día antes de su ingreso al hospital con cefalea intensa y diplopia. En la
exploración física se aprecio ptosis palpebral derecha, desviación del
ojo del mismo lado hacia fuera y anisocoria por dilatación pupilar
derecha sin respuesta a la luz. El par craneal que se encuentra
afectado en este paciente es el:

A II, izquierdo

B II, derecho

C III, derecho


El tratamiento de elección para los pacientes escolares que tienen
diabetes insípida central consiste en administrarles:

A Carbamazepina

B Espironolactona

C Desmopresina

D Líquidos

La intoxicación por nitratos en una niña de tres años requiere la
administración de:

A Hidralazina

B Piridoxina

C Azul de metileno

D Loratadina

El hallazgo del gen BCR1 es un factor de riesgo que se relaciona con el
cáncer de:

A Próstata

B Pulmón

C Colón

D Mama

Si se aplica la escala de Glasgow para evaluar el estado de conciencia
de una niña que sufrió traumatismo craneoencefalico, se debe
explorar:

A Respuesta motora, respuesta verbal y apertura ocular

B Respuesta al dolor, reflejo palpebral y respuesta verbal

C Respuesta pupilar, respuesta al dolor y respuesta motora

D Signo de Babinsky, apertura de los ojos y la respuesta motora

La coenzima-A se sintetiza a partir de:

A Vitamina A

B Ácido fólico

C Tiamina

D Ácido pantotenico

El grupo de antibióticos que produce osificación temprana del
cartílago de crecimiento en los niños es el de:

A Quinolonas

B Macrolidos

C Aminoglucosidos

D Cefalosporinas

En el lactante, la alimentación al seno materno debe complementarse
con:

A vitamina A

B Calcio

C Zinc

D Hierro

El antimicrobiano de elección para tratar a un lactante de 18 meses
de edad que presenta un segundo episodio de otitis media seis meses
después del primero es:

A Amoxicilina

B Amoxicilina/clavulanato

C Cefuroxima

D Ceftriaxona

El estudio más sensible para evaluar los padecimientos inflamatorios
crónicos de las estructuras nasosinusales es:

A Nasofibroscopia

B Ultrasonido

C Tomografía lineal

D Tomografía computada

La hipocalcemia que se presenta en la mayoría de los pacientes con
insuficiencia renal aguda es secundaria a:

A Alteraciones de la liberación de la hormona paratiroidea

B Acidosis metabólica

C Hiperfosfatemia

D Hipermagnesemia

El medicamento de elección para tratar el impétigo en un lactante de
ocho meses es:

A Amoxicilina

B Fosfomicina

C Dicloxacilina

D Amikacina

El objetivo de la cirugía antirreflujo en los pacientes que presentan
enfermedad por reflujo gastroesofagico es:

A Mejorar el vaciamiento gástrico

B Restablecer la función del esfínter esofágico superior

C Mejorar la contractilidad esofágica

D Mejorar la función mecánica del cardias

Una mujer de 50 años, que ha sido tratada con antiinflamatorios no
esteroideos, ingresa al servicio de urgencias por presentar
hematemesis muy abundante. El procedimiento más útil para iniciar
el tratamiento consiste en:

A Colocar un cateter venoso y reponer líquidos

B Practicar la esofagogastroscopia y administrar un inhibidor de la
bomba de protones

C Colocar una sonda nasogastrica y efectuar el lavado con agua
helada

D Administrar sangre total y realizar una serie esofagogastroduodenal

El tratamiento de elección de un recién nacido de 23 días con
diagnóstico de artritis séptica se debe efectuar con:

A Dicloxacilina y amikacina

B Ampicilina y gentamicina

C Ampicilina y cefotaxima

D Ceftriaxona y vancomicina

El esquema más útil para tratar la ulcera duodenal activa asociada a
Helicobacter pylori en los adolescentes consiste en administrar:

A Ranitida y esomeprazol

B Cefazolina y ranitidina

C Claritromicina y omeprazol

D Bismuto y famotidina

La anomalía citogenética que se presenta en la leucemia mielocitica
crónica es la:

A Deleción del cromosoma 14

B Translocación reciproca 9:22

C Translocación del encogen RAS

D Trisomia 21

La administración de eritromicina durante la fase paroxística de la tos
ferina tiene como objetivo:

A Disminuir los accesos de tos

B Evitar complicaciones respiratorias

C Disminuir la infección bacteriana secundaria

D Acortar el periodo de contagiosidad

El cuadro clínico de la intoxicación por organofosforados es causado
por:

A Aumento de catecolaminas

B Inhibición de la colinesterasa

C Aumento de la colinesterasa

D Aumento de la fosfodiesterasa

Para integrar el diagnóstico de glaucoma agudo por cierre angular se
debe encontrar:

A Dolor agudo, secreción mucopurulenta y miosis

B Dolor intenso, midriasis y reducción visual

C Dolor intenso, miosis y visión normal

D Dolor agudo, miosis e hipertensión ocular

La osteoporosis que presentan algunos pacientes que sufren de
síndrome de Cushing se debe a un:

A Aumento de la calcitonina

B Aumento de la acción de la vitamina D

C Estimulo de los osteoclastos

D Estimulo de la paratohormona

La defecación voluntaria inicia con:

A Incremento de la presión intraabdominal

B Relajación del esfínter anal externo

C Distensión del recto

D Rectificación de la angulación anorrectal

El fármaco de elección para iniciar el tratamiento de la colitis crónica
inespecífica moderada es:

A Sulfasalazina

B Mesalamina

C Infliximab

D Azatioprina

La malformación que puede asociarse con el polihidramnios es:

A Atresia de esófago

B Mielomeningocele

C Riñón poliquistico

D Hidrocefalia

El sitio más importante de producción de los elementos
hematopoyeticos en la etapa fetal es:

A Médula ósea

B Bazo

C Hígado

D Riñón

La complicación del infarto del miocardio que da lugar a la mitad de
las muertes durante la primera hora de evolución es:

A Bradicardia sinusal

B Fibrilación auricular

C Fibrilación ventricular

D Taquicardia sinusal

Para el tratamiento de la hiperplasia prostática benigna esta indicado
el uso de:

A Sildenafil

B Finasteride

C Flutamida

D Prazosina

Los antimicrobianos que actúan uniéndose a la subunidad ribosomal
50-S, lo que no permite la síntesis de las proteínas bacterianas,
corresponden al grupo de:

A Carbapenemicos

B Aminoglucosidos

C Tetraciclinas

D Macrolidos

En el tratamiento de los pacientes con vejiga neurogenica espastica,
es de utilidad la administración de:

A Flavoxato

B Escopolamina

C Oxibutina

D Atropina

El fármaco que actúa a nivel de los receptores plaquetarios es:

A Ticlopidina

B Urocinasa

C Fraxiparina

D Heparina

El antiviral de elección para el tratamiento de la coriorretinitis
causada por el citomegalovirus en un neonato de 20 días de nacido
es:

A Ganciclovir

B Aciclovir

C Ribavirina

D Amantadita

Entre las acciones fisiológicas de la insulina se encuentran la:


A Lipólisis y cetogenesis

B Lipogenesis y anabolismo

C Glucogenolisis y catabolismo

D Gluconeogenesis y glucolisis

El diurético de mayor utilidad para el manejo y control de la
hipertensión arterial en estadio I es la:

A Furosemida

B Espironolactona

C Bumetanide

D Clorotiazida

La neoplasia más sensible al metotrexate y la actinomicina D es:

A Enfermedad trofoblastica gestacional

B Disgeminoma ovárico

C Carcinoma mucinoso de ovario

D Carcinoma endometrial

El estudio más sensible para hacer el diagnóstico de ruptura
prematura de membranas es la:

A Cuantificación de líquido amniótico

B Cristalización de líquido amniótico

C Prueba con papel de nitrazina

D Presencia de células naranja en el líquido amniótico

Ante la aparición de dolor en el epigastrio e hipocondrio derecho en
una paciente que cursa con preeclampsia, el diagnóstico más
probable es:

A Síndrome de HELLP

B Hematoma subcapsular hepático

C Colecistitis aguda

D Inminencia de eclampsia

El medicamento que inicialmente se debe administrar ente la
presencia de tumefacción e hipersensibilidad mamaria en una
paciente con síndrome premenstrual es:

A Paracetamol

B Naproxen

C Danazol

D Trometanina

El fármaco utilizado para el tratamiento de la enfermedad de Ménière
y que previene el vértigo recurrente es:

A Meclicina

B Cinarizina

C Hidroclorotiazida

D Dimenhidranato

El tipo histopatológico más frecuente del cáncer de mama es el:

A Intracanalicular

B Ductal infiltrante

C Lobular invasor

D Intralobulillar

El agente farmacológico de elección para prevenir eventos convulsivos
en las pacientes que padecen preeclampsia severa es:

A Fenobarbital

B Diazepam

C Sulfato de magnesio

D Difenilhidantoina

La prueba específica para establecer el diagnóstico de diabetes
gestacional es:

A Determinación de péptido E

B Glucemia postcarga de 50 g

C Glucemia en ayuno

D Curva de tolerancia a la glucosa

La prueba que se emplea como procedimiento predictivo de parto
pretermino es la de:

A Flama

B Nitrazina

C Células naranja

D Fibronectina fetal

La mayor absorción de sodio se realiza en:

A Porción ascendente del asa de Henle

B Porción descendente del asa de Henle

C Tubulo proximal

D Tubulo distal

¿Cuál es el antimicrobiano más útil para el tratamiento del tétanos
neonatal?

A Cefotaxima

B Dicloxacilina

C Claritromicina

D Penicilina G

¿Cuál de las siguientes opciones hace sospechar hipotiroidismo en el
RN?

A Fontanela anterior puntiforme

B Suturas parietales imbricadas

C Talla menor de 45 cms

D Fontanela posterior amplia

¿Cuál es el estudio de mayor especificidad y sensibilidad para hacer el
diagnóstico de reflujo gastroesofágico?

A Endoscopia

B Manometria

C pH-metria

D Serie esofagogastroduodenal

¿Cuál es el medicamento de elección para tratar a un escolar que
presenta pneumonia por Micoplasma pneumoniae?

A Penicilina

B Ceftazidima

C Claritromicina

D Amoxicilina

En fase cefálica de la digestión en el niño, el estómago es estimulado
por el nervio:

A Olfatorio

B Glosofaríngeo

C Hipogloso

D Vago

Para penetrar en la célula, los iones hidrógeno se intercambian con:

A Magnesio

B Sodio

C Fósforo

D Calcio

¿Cuál es la vía principal de transmisión de las infecciones
nosocomiales?

A Hematógena

B Quirúrgica

C Urinaria

D Fomites

¿Cuál es el antimicótico de primera elección para tratar a un paciente
de 60 años que presenta meningitis criptococica?

A Griseofulvina

B Miconazol

C Anfotericina B

D Ketoconazol

La producción deficiente de una de las cadenas de globina, alfa o
beta, es el mecanismo que da origen a la:

A Talasemia

B Anemia perniciosa

C Anemia de las células falciformes

D Anemia megaloblastica

El beriberi se debe a deficiencia de:

A Niacina

B Ácido ascórbico

C Tiamina

D Riboflavina

En la leche humana se han encontrado anticuerpos contra:

A Escherichia coli, Salmonella sp, y Shiguella sp

B Varicela zoster, Herpes virus, y Pseudomonas Aeruginosa

C Helicobacter pylori, Pneumocistis carinii, y Escherichia coli

D Toxoplasma, Citomegalovirus, y Shiguella sp

Los pacientes que presentan bronquiectasias infectadas por
Pseudomonas aeruginosa deben ser tratados mediante el uso de:

A Linezolid

B Azitromicina

C Gatifloxacina

D Ciprofloxacina

En un neonato que en las primeras horas de vida presenta vómitos de
contenido biliar y cuya radiografía de tórax y abdomen muestra el
signo de la doble burbuja. El diagnóstico más probable es:

A Estenosis congénita de píloro

B Atresia duodenal

C Invaginación intestinal

D Hernia diafragmática

Un hombre de 32 años con diagnóstico de SIDA, tiene una cuenta baja
de linfocitos, inicia su padecimiento actual hace dos semanas con
visión borrosa y disminución de la agudeza visual de predominio
derecho. El diagnóstico clínico más probable es de retinitis por:

A Citomegalovirus

B Varicela zoster

C Toxoplasma

D Herpes simple

El dato clave para orientar el diagnóstico clínico en un paciente con
enfermedad vascular mesentérica es la presencia de:

A Dolor abdominal difuso

B Cianosis periumbilical

C Evacuaciones sanguinolentas

D Irritación peritoneal

El síndrome de cuello rojo se puede producir con la infusión rápida
de:

A Cefepime

B Fosfomicina

C Vancomicina

D Rifampicina
Hombre de 20 años con antecedente de alergia a multiples alimentos,
con manifestaciones cutaneas papulas y placas circunscritas
confluentes edematosas, eritematosas, erupcion ampollosa y fiebre de
3 dias, actualmente en tratamiento con penicilina por
faringoamigdalitis aguda

A Urticaria aguda

B Eritema multiforme

C Dermatitis atopica

D Queratitis
Mujer de 23 años presenta ataque al estado general, cefalea, rinorrea
hialina, asi como odinofagia de 1 semana evolucion, agragandose tos y
espectoracion hialina hace 5 dias, fiebre de 38.7, FC 110 lpm, FR 28
RPM, Ta 100/60, Saturacion al 92%, faringe hipermica, conjuntiva
hipermica, campos pulmonares estertores crepitantes diseminados
bilaterales y sibilancias difusas

A Neumonia por neumococ

B Neumonia por chlamydia

C Asma agudizada


Hombre de 42 años bajo ingesta de alcohol, sufre agresion por
terceras personas, golpes contusos en torax y abdomen, presenta
dolor a nivel hipogastrio, dificultad para evacuar, dolor a la miccion y
hematuria macroscopica, palidez de tegumentos, facies algica, sin
compromiso cardiorrespiratorio, palidez de tegumentos, abdomen
datos de irritacion peritoneal, por lo que pasa a laparotomia

A Lesion renal

B Lesion prostatica

C Lesion uretral


Mujer de 13 años traida por la amdre a urgencias por dolor
abdominal y fiebre. No recibiovacuna de BCG, inicia hace dos meses
con dolor abdominal generalizado y diarrea intermitente, hace un
mes fiebre nocturna de 39 tos y expectoracion, hiporexia perdiad de 9
kg de peso, examen laboratorio especificao para el diagnostico de
este caso

A Frotis de sangre periferica

B Rosa de Bengala

C BAAR en esputo


EL medicamento de elección para el tratamiento de las crisis
convulsivas neonatales es:

A Fenobarbital

B Fenitoina

C Diazepam

D Clonazepam

El signo de Trendelemburg en los pacientes que padecen luxación
congénita de la cadera, se debe a qué existe flacidez del músculo:

A Sartorio

B Vasto interno

C Psoas-iliaco

D Glúteo medio

El tratamiento de la fase aguda de la tripanosomiasis se debe llevar
mediante la administración de:

A Pirantel

B Benznidazol

C Eflornitina

D Nifurtimox

Un hombre de 34 años presenta una chancro ulcerado en el dorso de
la mano, qué forma una cadena de lesiones nodulares
eritemaviolaceas y no dolorosas, que siguen el trayecto de los vasos
linfáticos regionales del miembro torácico. El diagnóstico más
probable es:

A Nocardiosis

B Tuberculosis cutanea

C Micetoma

D Esporotricosis

Para hacer el diagnóstico electrocardiográfico de hipocalcemia en un
neonato de quince días, se debe encontrar:

A Intervalo Q-T alargado

B Espacio R-S corto

C Intervalo P-R alargado

D Inversión de la onda T

La presencia de leucocoria en la etapa neonatal sugiere la existencia
de:

A Catarata

B Retinoblastoma

C Dacrioestenosis

D Queratocono

La pelagra se debe a deficiencia de:

A Riboflavina

B Tiamina

C Piridoxina

D Niacina

La absorción de hierro se favorece mediante la administración de:

A Vitamina A

B Vitamina B6

C Vitamina B12

D Vitamina C

El medicamento de primera elección para tratar la colitis
pseudomembranosa en el paciente con diabetes mellitus tipo 2 es:

A Clindamicina

B Vancomicina

C Ceftazidima

D Metronidazol

La alteración determinante en la microesferocitosis hereditaria se
encuentra en:

A El bazo

B Las cadenas de globina

C El hígado

D La membrana del eritrocito

La profilaxis de la hepatitis A se debe realizar en todos los contactos
personales cercanos de los pacientes enfermos durante el periodo:

A Incubación

B Sintomático

C Ictérido

D Febril

La trisomia 21 suele asociarse con:

A El nefroblastoma

B El meduloblastoma

C La leucemia linfoblastica

D El tumor de células germinales

El marcador más útil para establecer el diagnóstico de cirrosis biliar
primaria es la determinación de anticuerpos anti:

A Nucleares

B Músculo liso

C Mitocondriales

D Microsomales

El signo radiológico de doble burbuja se encuentra en recién nacidos
que presentan:

A Hipertrofia Pilarica

B Invaginación intestinal

C Atresia duodenal

D Malrotación intestinal

El padecimiento que con mayor frecuencia puede asociarse con la
fibrosis quística es:

A Enterocolitis necrosante

B Íleo meconial

C Enfermedad de Hirschprung

D Hipotiroidismo

Para el tratamiento de la varicela den un lactante de 6 meses está
indicado utilizar:

A Aciclovir

B Ganciclovir

C Amantadina

D Ribavirina

La fibrosis quística es una enfermedad hereditaria qué se transmite de
manera:

A Recesiva ligada al sexo

B Dominante ligada al sexo

C Autosómica dominante

D Autosómica recesiva

La condición que con mayor frecuencia se relaciona con la anemia
megaloblastica en preescolares es:

A Sangrado digestivo crónico

B Desnutrición

C Uso prolongado de ácido acetilsalicílico

D Mala absorción intestinal

El sucralfato ejerce su acción terapéutica debido a que:

A Es un potente bloqueador H2

B Protege el lecho de la ulcera

C Disminuye la fase cefálica de la secreción gástrica

D Aumenta la alcalinidad gástrica

El agente etiológico qué más frecuentemente produce artritis séptico
en pre-escolar es:

A Staphylococcus epidermidis

B Staphylococcus aureus

C Streptococcus pyogenes

D Streptococcus pneumoniae

A diferencia del ADN, en el ARN la timina ha sido reemplazada por:

A Adenina

B Citosina

C Uracilo

D Ribosa

El diagnóstico definitivo de megacolon congénito se hace mediante:

A Estudio histopatológico

B Ultrasonido abdominal

C Enema baritado

D Manometría rectal

El aminoácido precursor de la biosíntesis de catecolaminas es la:

A Tirosina

B Arginina

C Histidina

D Taurina

A los adolescentes tuberculosos tratados con isoniazida se les debe
administrar en forma complementaria:

A Tiamina

B Piridoxina

C Ácido Ascórbico

D Ácido fólico

Las células de Reed-Sternmerg se observan específicamente en casos
de:

A Leucemia linfoblástica

B Neuroblastoma

C Linfoma de Hodgkin

D Linfoma no Hodgkin

El medicamento que se asocia con la reducción del riesgo de cáncer
colorrectal es:

A Ranitidina

B Verapamil

C Ácido acetilsalicílico

D Metoclopramida

La anormalidad citogenética que se presenta en la leucemia
mielocitica crónica es la:

A Deleción del cromosoma 14

B Translocación reciproca 9:22

C Translocación del oncogen-RAS

D Trisomia 21

En la mayoría de los casos, el diagnostico citogénetico del síndrome
de Turner corresponde a la formula cromosómica:

A 46 XX, con X frágil

B 45X

C 46XX, con X anular

D 47XXX

El mecanismo por el cual la acarbosa ejerce si acción es:

A Estimula la liberación de insulina

B Aumenta la sensibilidad a la insulina

C Disminuye la gluconeogenesis

D Inhibe la alfa-glucosidasa intestinal

El fármaco que debe evitarse en los pacientes qué padecen de gota
por su capacidad para inhibir la excreción urinaria del ácido úrico es:

A Irbesartan

B Bezafibrato

C Hidroclorotiazida

D Alprazolam

En el tratamiento de la hipertensión crónica en mujeres embarazadas
esta contraindicado el uso de:

A Metildopa

B Hidralazina

C Captopril

D Labetadol

El diagnóstico de artritis gotosa aguda se confirma si:

A Los niveles de ácido úrico en el suero están elevados

B Se observa condrocalcinosis en la articulación afectada

C La primera articulación metatarsofalangica esta afectada
clínicamente

D Se observan cristales birrefringentes en el líquido sinovial

La enzima cuya deficiencia en los eritrocitos es el factor más
frecuentemente responsable de la anemia hemolítica es la:

A Piruvatocinasa

B Hexocinasa

C Glucosa-6-fosfato deshidrogenasa

D Adenilatociclasa

Para hacer el diagnóstico de hemorroides grado II es preciso
encontrar:

A Sangrado sin prolapso

B Sangrado con prolapso

C Prolapso y trombosis

D Prolapso con reducción espontánea

El efecto farmacológico del alopurinol se debe a que:

A Inhibe la xantina-oxidasa

B Solubiliza el ácido úrico

C Inhibe la fosforribosil-aminotransferasa

D Acelera la degradación del ácido úrico

La vacuna pentavalente protege contra:

A Haemophilus no capsulados, hepatitis B, polio, sarampión y varicela

B Tos ferina, difteria, tétanos, sarampión y Haemophilus influenzae
tipo B

C Haemophilus influenzae tipo B, hepatitis A, difteria, tos ferina y
tétanos

D Difteria, tos ferina, tétanos, hepatitis B y Haemophilus influenzae
tipo B

Las dosis de DPT que ya debiera haber recibido un preescolar de 5
años sano son:

A 1 y 2 refuerzos

B 2 y 1 refuerzo

C 2 y 2 refuerzos

D 3 y 2 refuerzos

Los antimicrobianos qué actúan uniéndose a la subunidad ribosomal
50-S, lo que no permite la síntesis de las proteínas bacterianas,
corresponden al grupo de:

A Carbaperemicos

B Aminoglucosidos

C Tetraciclinas

D Macrolidos

Para tratar la taquicardia ventricular que produce compromiso
hemodinámico esta indicado el uso de:

A Digital

B Procainamida

C Desfibrilación

D Adenosin

Un hombre de 22 años acude a consulta por un padecimiento de una
semana de evolución manifestado por fiebre, dificultad para caminar
y subir escalones, perdida proximal de la fuerza muscular, lesiones
dérmicas en la cara y en la “V” del cuello. Al explorarlo, la fuerza
muscular se encuentra se encuentra en 3 de 5 y se observa rash “en
heliotropo” en los parpados. Los exámenes de laboratorio muestran:
CPK, 1200 U/DL y DHL, 789 U/DL. El diagnóstico clínico más probable
es:

A Mielitis transversa

B Esclerodermia

C Dermatomiositis

D Enfermedad de Guillain-Barré
Para establecer el diagnóstico de certeza, se debe practicar:

A Determinación de aldolasa serica

B Electromiografia

C Biopsia de músculo

D Estudio de LCR

La complicación aguda de la diabetes que aparece como resultado de
una deficiencia total de insulina e incremento de glucagón es:

A Coma hiperosmolar no cetosico

B Cetoacidosis

C Hipoglucemia

D Estado hiperosmolar

Una mujer de 45 años alérgica al ácido acetilsalicílico, acude a
consulta por presentar un cuadro de artritis simétrica de las pequeñas
articulaciones, acompañada de rigidez matutina de mas de 1 hora de
evolución. El medicamento que se debe utilizar como inductor de la
remisión reumatológica es:

A Indometacina

B Metotrexate

C Sulindac

D Prednisona

Para su replicación y expresión, el virus de la hepatitis D requiere la
coinfección del virus de la hepatitis:

A A

B B

C C

D E

En los pacientes que padecen espondilitis anquilosante
frecuentemente se encuentra:

A Velocidad de eritrosedimentación reducida

B Factor reumatoide positivo

C Leucopenia

D Antigeno HLA-B27 positivo

Cuando existe epistaxis anterior, la hemorragia procede de:

A Arteria etmoidal anterior

B Arteria nasal interna

C Plexo de Kisselbach

D Arteria esfenopalatina

La afinidad de la hemoglobina por el oxigeno disminuye al:

A Disminuir la PCO2

B Disminuir la concentración de H

C Aumentar la concentración de 2,3 difosfoglicerato

D Aumenta la concentración de bicarbonato

El mejor tratamiento para los pacientes que padecen anemia por
esferocitosis hereditaria es:

A Administración de eritropoyetina

B Administración de ácido fólico

C Administración de vitamina B12

D Esplenectomía

La manifestación más frecuente del mieloma múltiple en los estudios
de química sanguínea es la:

A Hipernatremia

B Hiperuricemia

C Hiperfosfatemia

D Hipercalcemia

La intoxicación aguda por benzodiacepinas en los adolescentes se
debe tratar mediante la administración de:

A Flumazenil

B Atropina

C Bicuculina

D Naloxona

Un hombre de 23 años es traído al servicio de urgencias por haber
ingerido diazepam en dosis desconocida y con fines suicidas. El
paciente se encuentra ya con soporte vital básico asegurado. El
antídoto específico en este caso es:

A Flumazenil

B Naloxona

C Carbón activado

D Pracidocina

El medicamento que se administra al mismo tiempo que la isoniazida
para evitar el desarrollo de neuropatía periférica es:

A Cianocobalamina

B Alfa-tocoferol

C Riboflavina

D Piridoxina

El tratamiento de la fase aguda de la tripanosomiasis se debe llevar a
cabo mediante la administración de:

A Pirantel

B Pirimetamina

C Pentamidina

D Nifurtimox

EL tratamiento de elección para la meningitis pneumococica en los
lactantes consiste en combinar:

A Penicilina y cloranfenicol

B Ceftriaxona y vancomicina

C Ampicilina y cefotaxima

D Ceftazidima y rifampicina

El fármaco de primera elección para el tratamiento de la enuresis es
la:

A Disipramina

B Paroxetina

C Desmopresina

D Carbamazepina

En los casos de pancreatitis aguda grave disminuye la concentración
sérica de:

A Triglicéridos

B Calcio

C Amilasa

D Aspartato-aminotransferasa

Los microorganismos que con mayor frecuencia son causa de la
neumonía nosocomial son:

A Haemophilus influenzae y Moraxella catarrhalis

B Legionella pneumophyla y Candida albicans

C Moraxella catarrhalis y Streptococcus pneumoniae

D Pseudomonas aeruginosa y Klebsiella pneumoniae

El antibiótico de elección para tratar a un escolar que padece
neumonía causada por Micoplasma pneumoniae es:

A Teicoplanina

B Lincomicina

C Ceruroxima

D Claritromicina

La fibrosis quística se hereda en forma:

A Autosómica dominante

B Autosómica recesiva

C Dominante ligada al cromosoma X

D Dominante ligada al cromosoma Y

La presencia de antígeno de histocompatibilidad HLA-B27 se asocia
con:

A Artritis reumatoide

B Espondilitis anquilosante

C Esclerodermia

D Dermatomiositis

En la esquizofrenia los trastornos senso-perceptivos más frecuentes
son las alucinaciones:

A Táctiles

B Visuales

C Auditivas

D Olfatorias

El diagnóstico de diabetes insípida se debe establecer ante la
presencia de:

A Poliuria, densidad urinaria baja e hipernatremia

B Oliguria, densidad urinaria alta e hiponatremia

C Poliuria, densidad urinaria normal y eunatremia

D Poliuria, densidad urinaria alta e hipernatremia

En el estudio citoquímico del LCR de un lactante de un año, los datos
que apoyan el diagnóstico de meningitis tuberculosa son:

A Hiperproteinorraquia y normoglucorraquia

B Pleocitosis y normoglucorraquia

C Hiperproteinorraquia e hipoglucorraquia

D Hipoglucorraquia y cifras normales de proteínas

Se puede presentar hipotiroidismo como reacción adversa de la
administración prolongada de:

A Carbamacepina

B Clonazepam

C Amitriptilina

D Litio

El tratamiento de la enfermedad celiaca en el lactante consiste en:

A Suprimir el gluten de la dieta

B Proporcionar una dieta libre de lactosa

C Administrar colestiramina

D Administrar neomicina

El medicamento de elección para llevar a cabo la profilaxis en los
contactos domiciliarios de pacientes con enfermedad menigococica
es:

A Cloranfenicol

B Rifampicina

C Tetraciclina

D Dixiciclina

En el hipotiroidismo congénito primario se encuentra:

A Tiroxina baja y tirotropina alta

B Tiroxina alta y tirotropina baja

C Tiroxina baja y tirotropina normal

D Tiroxina normal y tirotropina baja

El procedimiento quirúrgico más adecuado para el tratamiento de la
fisura anal crónica es la:

A Anaplastia

B Esfinterotomia lateral interna

C División del músculo esfínter anorrectal

D Fistulotomia

El fármaco idóneo para el tratamiento del hipotiroidismo congénito
es:

A Extracto tiroideo

B Levotiroxina

C Tirotrofina

D Tiroxina

El medicamento de primera elección para el tratamiento de la
shiguellosis en escolares es:

A Furazolidona

B Gentamicina

C Ampicilina

D TMP-SMX

El marcador para establecer el diagnostico de cirrosis biliar primaria
es la determinación de anticuerpos anti:

A Nucleares

B Músculo liso

C Mitocondriales

D KLM1

La afinidad de la hemoglobina por el oxigeno esta modulada por la
concentración de:

A 2,3-difosfoglicerato

B Difosfato de nucleosido

C Fosfato de piridoxal

D Fosfoenolpiruvato

El diagnóstico de certeza de la tromboembolia pulmonar se establece
por medio de:

A Resonancia magnetica nuclear

B Gammagrama pulmonar ventilatorio perfusorio

C Pruebas de función respiratoria

D Angiografía pulmonar

El tratamiento de la anemia perniciosa se debe llevar a cabo mediante
la administración de:

A Niacina (B3)

B Riboflavina (B2)

C Ácido fólico

D Hidroxicobalamina (B12)

El tratamiento de elección de la meningitis bacteriana en pacientes
adultos consiste en la administración combinada de dexametasona y:

A Cefalexina + ciprofloxacina

B Ceftazidima + vancomicina

C Ceftriaxona + amikacina

D Cefepime + gentamicina

El agente causal mas frecuente en la endocarditis infecciosa en
pacientes consumidores de drogas administradas por vía intravenosa
es:

A Aspergillus fumigatus

B Pseudomonas aeruginosa

C Klebsiella pneumoniae

D Staphylococcus aureus

El medicamento de elección para hacer la profilaxis de la diarrea del
viajero es:

A Ciprofloxacino

B TMP-SMX

C Tetraciclina

D Cefalexina

La mayor parte de los genes del complejo mayor de
histocompatibilidad se localiza en el cromosoma:

A 2*

B 6*

C 10*

D 18*

A una paciente de 26 años que padece una tormenta tiroidea se le
debe administrar:

A Yodo radioactivo

B Metilprednisolona

C Propiltiouracilo

D Levotiroxina

El tratamiento de la urgencia hipertensiva se debe efectuar mediante
la administración de:

A Captopril

B Hidralazina

C Fenodolpam

D Nitroprusiato de sodio

La osmolaridad sérica puede ser calculada si se conocen los valores
séricos de sodio, potasio y:

A Hematocrito y bicarbonato

B Nitrogeno ureico y glucosa

C Glucosa y cloro

D Bicarbonato y urea

El mecanismo por el que se produce el fenómeno de Raynaud es de
tipo:

A Vasogenico

B Cardiogenico

C Neurogenico

D Metabólico

En la mayoría de los casos, diagnóstico de anemia hemolítica
autoinmune puede establecerse por medio de la:

A Determinación de C3 y C4

B Cuantificación de inmunoglobulinas

C Prueba de Coombs directa

D Prueba de Coombs indirecta

El tumor testicular maligno más frecuente es el:

A Seminoma

B Carcinoma de células de Leydig

C Coriocarcinoma

D Teratocarcinoma

El medicamento más útil para tratar la infección pélvica aguda grave
producida por el gonococo es la:

A Penicilina

B Gentamicina

C Ceftriaxona

D Clindamicina

La primera manifestación de la nefropatía diabética en los estudios de
laboratorio es:

A Disminución del índice de filtración glomerular

B Aumento progresivo de la creatinina sérica

C Disminución de la excreción de la cistatina C

D Presencia de microalbuminuria

El antihipertensivo cuyo uso se debe evitar cuando existe la sospecha
clínica de que el paciente tiene hipertensión renovascular es el:

A Metoprolol

B Diltiazem

C Labetadol

D Captopril

El daño renal causado por los antiinflamatorios no esteroideos se
debe a:

A El aumento de la producción de óxido nítrico

B El bloqueo de la producción de prostaglandinas

C La inhibición de los leucotrienos

D La producción elevada de glucoproteina IIb/IIIa

La causa más frecuente de muerte por infarto del miocardio es:

A Arritmia

B Insuficiencia cardiaca

C Disfunción del músculo papilar

D Bloqueo auriculoventricular

El tipo de pólipo colonico que con mayor frecuencia se relaciona con
el cáncer de colon es el:

A Adenomatoso

B Velloso

C Juvenil

D Inflamatorio

La afección extraesqueletica más frecuente en los casos de
espondilitis anquilosante es la:

A Renal

B Neurológica

C Dermatológica

D Cardiovascular Oftálmica

En los pacientes qué padecen anemia ferropriva se encuentran cifras:

A Bajas de hierro y capacidad de fijación baja

B Bajas de hierro y capacidad de fijación elevada

C Altas de hierro y capacidad de fijación elevada

D Altas de hierro y capacidad de fijación baja

La producción de anticuerpos IgG contra las glucoproteínas de la
membrana plaquetaria es el mecanismo fisiopatologico de la púrpura:

A Trombocitopenica trombotica

B Trombocitopenica idiopática

C Por esteroides

D Vascular

El esteriode de elección para tratar la crisis addisoniana es la:

A Dexametasona

B Hidrocortisona

C Betametasona

D Prednisolona

Para el tratamiento del fenómeno de Raynaud se debe utilizar:

A Diltiazem

B Ácido acetilsalicilico

C Nifedipino

D Hidralazina

El crecimiento cardiaco que ocurre en los pacientes que padecen cor-
pulmonale crónico se inicia en:

A Aurícula izquierda

B Ventrículo derecho

C Aurícula derecha

D Ventrículo izquierdo

La vía principal de transmisión del Toxoplasma gondii es:

A Digestiva

B Hematógena

C Urinaria

D Respiratoria

Un ejemplo de bloqueador beta-adrenergico cardioselectivo es:

A Prazosina

B Metoprolol

C Propanolol

D Reserpina

Un hombre de 56 años, a quien se le practicó una gastrectomía parcial
hace 4 años, se presenta a consulta por padecer un síndrome
anémico. El laboratorio informa: hemoglobina, 5.2 gr/dl; HTO, 17%;
volumen corpuscular medio, 120; y concentración media de
hemoglobina, 32. El tratamiento a largo plazo y más adecuado para
este paciente consiste en administrarle:

A Paquetes globulares periódicamente

B Hierro por vía parenteral

C Hierro por vía oral


En los pacientes que padecen de osteodistrofia renal se encuentra:

A Disminución del fosforo sérico

B Elevación de la hormona paratiroidea

C Elevación del calcio sérico

D Elevación de la calcitonina

La causa más frecuente síndrome nefrótico en sujetos adultos es la:

A Glomerulonefritis de cambios mínimos

B Glomerunefritis focal y segmentaría

C Glomeruloesclerosis

D Glomerulopatia membranosa

En un recién nacido con tetralogía de Fallot se encuentra:

A Hipertrofia del ventrículo izquierdo, cabalgamiento de la aorta,
obstrucción del tracto de salida del ventrículo derecho y PCA

B Comunicación interventricular, estenosis infundibular de la
pulmonar, hipertrofia del ventrículo derecho y comunicación
interauricular

C Comunicación interventricular, estenosis infundibular de la
pulmonar, cabalgamiento de la aorta e hipertrofia del ventrículo
derecho


Un agricultor de 38 años que vive en una casa que tiene paredes y
techo de teja y piso de tierra, inicio su padecimiento actual con
edema palpebral y periocular unilateral, indoloro, fiebre y anorexia;
posteriormente se agregaron edema en la cara y en los miembros
inferiores, y síntomas gastrointestinales. Los estudios de gabinete
muestran megaesófago, megacolon y bloqueo de la rama derecha del
haz de his. El diagnóstico probable es:

A Tripanosomiasis

B Leishmaniasis

C Oncocercosis

El factor reumatoide reacciona contra la porción:

A FC de la IgA

B FAB de la IgE

C FC de la IgG

D FAB de la IgM

En las mujeres mayores de 70 años, la osteoporosis se manifiesta más
frecuentemente en:

A Humero, tobillo y cadera

B Tercio distal de radio, cadera y vértebras

C Cubito, codo y fémur

D Clavícula, codo y muñeca

Los pacientes que sufren insuficiencia suprarrenal primaria presentan:

A Hipercalcemia, hiponatremia e hiperpotasemia

B Hipocalcemia, hipernatremia e hipopotasemia

C Hiperpotasemia, hipermagnesemia e hiponatremia

D Hiperpotasemia, hipocalcemia e hiponatremia

El patrón serologico que establece el diagnóstico de hepatitis B aguda
es:

A Anti-HBc (-) y HBsAg (+), Anti-HBs (-), Anti-HBc ZgM (+) y HBeAg (+)

B Anti-HBc (-) y HBsAg (+), Anti-HBs (-), Anti-HBc Zg6 (+) y HBeAg (+)

C Anti-HBc (-) y HBsAg (+), Anti-HBs (-), Anti-HBc Zg6 (+) y HBeAg (-)

D Anti-HBc (-) y HBsAg (-), Anti-HBs (+), Anti-HBc (-) y HBeAg (-)

La enfermedad hereditaria que con mayor frecuencia se relaciona con
el adenocarcinoma de colon es:

A Poliposis gastrointestinal difusa

B Poliposis juvenil múltiple

C Síndrome de adenomas planos

D Poliposis adenomatosa

La trisomia 21 suele asociarse con:

A Nefroblastoma

B Meduloblastoma

C Leucemia linfoblástica

D Tumor de células germinales

El medicamento de elección para reducir la magnitud de la poliuria y
la polidipsia de los pacientes que padecen diabetes insípida es la:

A Corticotropina

B Espironolactona

C Desmopresina

D Clorpropamida

El dato de laboratorio que permite fundamentar el diagnóstico de
nefropatía diabética incipiente es la:

A Microalbuminuria

B Microhematuria

C Acidosis tubular renal

D Elevación ligera de la creatinina plasmática

Lo más frecuente es que las neoplasias malignas del intestino grueso
se localicen en el:

A Ciego

B Colon ascendente

C Colon transverso

D Sigmoides

El síndrome carcinoide es consecuencia de la secreción anómala de:

A Péptido intestinal vasoactivo

B Serotonina

C Noradrenalina

D Epinefrina

De acuerdo a la OMS la solución para rehidratación oral debe
contener:

A Glucosa, sodio, potasio, cloro y citrato

B Sodio, calcio, bicarbonato de sodio, magnesio y citrato

C Glucosa, calcio, sodio, potasio y citrato

D Glucosa, magnesio, cloro, potasio y citrato

El antiviral de elección para el tratamiento de la coriorretinitis
causada por el citomegalovirus humano es:

A Ganciclovir

B Aciclovir

C Ribavirina

D Vidaravina

Para hacer el diagnóstico de insuficiencia suprarrenal crónica se debe
encontrar:

A Debilidad, hipertensión arterial e hipokalemia

B Hiperpigmentación, hipotensión ortostatica e hiperkalemia

C Hipokalemia, hipotensión ortostatica e hiponatremia

D Hiperpigmentación, hiperglucemia e hipokalemia

En el estudio de LCR de los niños que padecen síndrome de Guillain-
Barré se encuentran:

A Células elevadas y glucosa baja

B Células normales y proteínas elevadas

C Proteínas normales y glucosa baja

D Glucosa normal y células elevadas

En una mujer que ingirió hace 2 horas una cantidad muy grande de
tabletas de clonazepam, el tratamiento más indicado consiste en la
administración de:

A Adrenalina

B Flumazenil

C Naltrexona

D Naloxona

Para que la hormona tiroidea lleve a cabo su función a nivel periférico
se requiere la conversión de:

A Triyodotironina a tiroxina

B Tiroglobulina a tiroxina

C Tiroxina a triyodotironina

D Hormona estimulante del tiroides a tiroxina

El inmunosupresor de elección adecuado para el tratamiento de la
nefritis lupica en adolescentes es:

A Metotrexate

B Ciclofosfamida

C Azatioprina

D Ciclosporina

La heparina es un polímero de mucopolisacarido que actúa uniéndose
a la:

A Proteína C

B Tromboplastina histica

C Proteína S

D Antitrombina III

Hombre de 51 años de edad con diagnóstico reciente de fractura de
fémur. Para completar su tratamiento requerirá de inmovilización por
los siguientes dos meses. La siguiente valoración NO ha demostrado
ser de utilidad para predecir el riesgo de desarrollar ulceras por
presión:

A valoración del estado nutricional

B exposición a la humedad

C movildad

D valoración vascular

Hombre de 43 años de edad que posterior a procedimiento quirúrgico
dental presenta alergia al látex. A partir de este momento el paciente
deberá evitar consumir el siguiente alimento por su alta asociación
con la alergia al latex:

A plátano

B trigo

C fresas

D huevo

Mujer de 29 años acude a consulta por presentar cefalea intensa
intermitente desde hace 6 meses. Usted considera el diagnóstico de
migraña. ¿Cuál es el factor de riesgo que debe evitar la paciente?

A dormir más de 6 horas al día

B los estímulos luminosos de la televisión

C las grasas insaturadas

D fibra abundante en la dieta

Masculino de 38 años, médico de profesión. Desde hace 2 meses
presenta sintomatología a nivel del sistema nervioso, sospechando
actualmente en esclerosis múltiple. Realiza usted punción lumbar
solicitando electroforesis de proteínas. ¿Cuál es la inmunoglobulina
que se encuentra elevada en el líquido cefalorraquideo de los
pacientes con esclerosis multiple?

A A

B D

C E


Se confirma el diagnóstico de sospecha, presentando actualmente un
ataque agudo. ¿Cúal es el tratamiento?

A plasmaferesis

B acetato de glatiramer

C interferon

D glucocorticoides

Hombre de 53 años de edad quien desde hace 3 meses presenta
anorexia, pérdida de peso y fatiga. Desde hace 3 semanas presenta
epistaxis y gingivorragia recurrentes. A la exploración física se
observan equimosis en todo el cuerpo y linfadenopatías en axilas y
región inguinal. La biometría hemática muestra leucopenia y los datos
histológicos sugieren leucemia mieloide aguda. ¿Cuál de las siguientes
opciones se relaciona directamente con este tipo de leucemia?

A exposición a benceno

B intoxicación crónica por plomo

C alcoholismo crónico

Mujer de 53 años de edad actualmente en estudio por probable
enfermedad pulmonar obstructiva crónica. ¿Cuál es estudio de
elección para realizar el diagnóstico?

A gasometría

B radiografía de tórax

C espirometría

D tomografía computarizada de tórax

Usted confirma el diagnóstico de sospecha. ¿Cuál de los siguientes
medicamentos son considerados los más importantes en el manejo
sintomático del paciente con EPOC?

A broncodilatadores

B mucolíticos

C oxígeno

D esteroides

Mujer de 21 años de edad que acude a la consulta externa al
presentar dermatosis, aparentemente secundaria a la administración
de amoxicilina. La paciente quiere saber cuál es su riesgo de padecer
una reacción cutánea severa. ¿Cuál es el porcentaje de las reacciones
adversas cutáneas severas secundarias a tratamiento farmacológico?

A 2%*

B 6%*

C 12%*

D 18%*

Mujer de 43 años con patología del sueño en estudio. Se indica
electroencefalograma y monitorización durante el sueño y en
despierto. entre los resultados se deberá observar que en la etapa del
sueño se observan movimientos oculares rápidos que en el adulto se
acompaña de:

A tono muscular elevado

B predominio del sistema parasimpático

C variaciones de la frecuencia cardíaca

D aumento de la presión arterial

Hombre de 62 años con diagnóstico reciente de enfermedad
pulmonar obstructiva crónica. ¿Cuál es el hallazgo fisiopatológico más
común que esperaría encontrar en este paciente?

A hipoxemia persistente y sintomática

B reducción persistente en la capacidad vital forzada

C reducción del volumen espirado máximo en el primer segundo de
la espiración forzada

D hipercápnea transitoria asintomática

Hombre de 19 años de edad, que acude con resultado de
coproparasitoscópicos en serie con quistes y trofozoitos de
entamoeba histolytica. Refiere antecedente de periodos alternos de
diarrea y estreñimiento. ¿Cuál es el tratamiento de elección indicado
en este paciente?

A quinfamida

B tinidazol

C etofamida

D metronidazol

Mujer de 40 años de edad, con antecedente de psoriasis desde hace 5
años. Acude a la consulta por presentar sinovitis en ambos codos. Se
realiza punción articular enviándose a estudio histopatológico el
líquido obtenido por la sospecha de artritis psoríasica. En caso de
confirmar el diagnóstico, ¿Cuál es el patrón de producción
característico de citocinas que se espera encontrar?

A interferón alfa

B interferón beta

C factor de necrosis tumoral alfa


Mujer de 70 años, hospitalizada desde hace 24 hrs con sospecha de
cáncer del cuerpo de páncreas. ¿Cuál de las siguientes opciones se
presenta en el 100% de los pacientes con cáncer del cuerpo del
páncreas?

A pérdida del peso

B dolor abdominal o persistente de espalda baja

C anorexia

D ictericia

¿cuál es es estudio que sirve para confirmar el diagnóstico?

A niveles de marcador tumoral ca 19-9

B análisis de cepillado y de jugo pancreático obtenido
endoscopicamente

C ultrasonido de páncreas, hígado y vía biliar

D tomografía helicoidal en fase arterial y portal

Mujer de 60 años con angina crónica estable. Se le realiza cateterismo
cardíaco encontrando lesión en coronaria descendente anterior, por
lo que se realiza angioplastía y colocación de stent. Dos horas después
del procedimiento refiere dolor precordial. A la exploración se
encuentra hipotensión arterial, taquicardia, taquipnea, pulso
paradójico e ingurgitación yugular. No se detectan soplos. Se toma
electrocardiograma que muestra complejos alternantes. ¿Cuál es el
diagnóstico clínico más probable?

A trombosis aguda del stent

B tamponade cardiaco


¿Cuál es el manejo que deberá establecerse de inmediato en este
paciente?

A cateterismo terapéutico

B inhibidores iib/iiia y heparina no fraccionada

C trombolisis y vasopresores

D drenaje pericárdico

Mujer de 46 años que ingreso al hospital con diagnóstico de
encefalitis en estudio. Actualmente se sospecha la posibilidad de
cursar con lupus eritematoso generalizado. ¿Cuáles son los
anticuerpos más sensibles que se deberán solicitar para el diagnóstico
de lupus eritematoso sistémico?

A antihistona

B antimicrosomales

C antinucleares

D antifosfolípidos

Masculino de 25 años de edad, de oficio pescador, acude con
sintomatología compatible con un síndrome anémico severo. Durante
su estudio usted descarta etiología carencial o neoplásica. ¿Cuál es la
parasitosis más probablemente asociada a la etiología de la anemia
del paciente?

A uncinariasis

B ascariasis

C trichuriasis

D triquinosis

Mujer de 32 años, en situación de calle, en quién usted sospecha la
posibilidad de anemia por deficiencia de hierro. ¿Cómo podría
confirmar el diagnóstico?

A sideremia incrementada

B disminución de los niveles de ferritina sérica

C disminución en la capacidad de fijación de hierro

D incremento en la saturación de transferrina

Hombre de 38 años de edad con diagnóstico reciente de trastorno por
somatización. ¿Cuál es la conducta médica indicada en este tipo de
pacientes?

A iniciar psicoterapia cognitivo- conductual

B informar al paciente que su problema es psicológico

C iniciar con ansiolíticos

D manejo con medicina alternativa

Hombre de 18 años con diagnóstico de acne severo, acude a su
consulta al presentar desde hace 1 semana sequedad de la piel,
epistaxis, conjuntivitis, fotosensibilidad y pérdida capilar. Usted
sospecha que se trata de efectos secundarios al tratamiento que esta
utilizando actualmente para el acné. ¿Cuál es el medicamento que
explica la aparición de estos efectos secundarios?

A Minociclina

B Peróxido de benzoilo

C Isotretinoína


Mujer de 18 años de edad, alérgica a la penicilina, con infecciones
respiratorias de repetición. Hace una semana presenta lesiones en
piel de tipo macular en media luna confluentes que aparecen y
desaparecen de forma espontánea. Se solicita exudado faríngeo en
búsqueda de estreptococo beta hemolítico el cual resulta positivo.
¿Cuál es la asociación clínica con el siguiente signo que sustenta el
diagnóstico de fiebre reumática en esta paciente?

A Poliartralgias

B Corea de sydenham

C Prolongación reversible del segmento PR

¿Cuál es el tratamiento de elección para erradicar el estreptococo en
esta paciente?

A penicilina g benzatínica

B azitromicina

C cefalexina

D amoxicilina

Mujer de 53 años, sedentaria y obesa. Presenta dolor precordial
súbito e intenso, opresivo, irradiado a extremidades superiores y
cuello desde hace 4 horas. Se toma electrocardiograma que reporta
ritmo sinusal con frecuencia de 110/min, desnivel positivo franco del
segmento ST en VI a V3. ¿Cuál es el proceso fisiopatológico que
explica el cuadro clínico del paciente?

A embolia

B oclusión ateromatosa

C trombosis in situ


¿cuál es la localización de la lesión cardíaca más probable?

A lateral alto

B anterior extenso

C inferior

D anteroseptal

Durante su estabilización, el paciente presenta el siguiente
electrocardiograma con ausencia de ondas P, intervalos RR irregulares
y ondas F, además de presentar hipotensión. ¿cuál es el tratamiento
que debe indicar?

A marcapasos

B cardioversión

C verapamil

D metoprolol

Masculino de 25 años de edad, con antecedente de sinusitis crónica.
Inicia su padecimiento actual hace 3 días con fiebre y malestar
general, el día de hoy ingresa a urgencias con severo deterioro
neurológico y un síndrome infeccioso. Se sospecha una meningitis
bacteriana. Las manifestaciones físicas presentes son
fundamentalmente secundarias a:

A Respuesta inmune del huésped

B Patogenicidad del microorganismo

C Tiempo de evolución de la enfermedad


¿Qué podriamos encontrar en la PL?

A leucocitos y proteínas bajas con glucosa normal

B leucocitos y proteínas elevadas con glucosa baja

C leucocitos y proteínas bajas con glucosa normal

D leucocitos y glucosa elevados con proteínas bajas

Hombre de 65 años, que acude a su consultorio al presentar cifra
tensional detectada en casa de 170/110 mm/hg. Usted corrobora la
cifra y no encuentra más datos patológicos. Usted decide dar
tratamiento farmacológico en su consultorio, esperando disminuir la
presión en las siguientes:

A 1 - 2 horas

B 2 - 6 hrs

C 8 - 12 hrs

D 24 - 48 hrs

¿cuál es el medicamento de elección para iniciar tratamiento
ambulatorio por vía oral?

A losartán

B metildopa

C nitroprusiato

D hidralazina

Durante la monitorización la presencia del siguiente dato obligaría a
trasladar al paciente a un segundo o tercer nivel de atención:

A tos

B epistaxis

C fosfenos

D disnea

Masculino de 58 años de edad, alcohólico desde hace 25 años. Desde
hace 15 días ha consumido diariamente más de un litro de alcohol.
Desde hace 2 días está en una granja para su desintoxicación. El día
de hoy comienza con temblor en las manos, ansiedad e insomnio
total. A la exploración se le encuentra desorientado, ansioso, confuso,
delirante, irritable y con tendencia a la agresividad. ¿Cuál es el
diagnóstico clínico más probable?

A encefalopatía de wernicke

B encefalopatía hepática

C esquizofrenia orgánica

Hombre de 19 años de edad, que es llevado a su consulta con
aparentes trastornos del comportamiento secundarios a la inhalación
de cocaína. Los efectos de esta sustancia sobre el comportamiento de
la persona que la consume es debido a:

A disminuye los niveles de glutamato

B inhibe la producción de gabapentina

C bloquea los canales de calcio y la captura de neurotransmisores

D bloquea la recaptura de dopamina y serotonina

Masculino de 25 años de edad, que acude al servicio de urgencias al
presentar desde hace 7 días evacuaciones disminuidas de consistencia
con moco y sangre, además de dolor abdominal difuso. Con la
exploración física sospecha la posibilidad de colitis amibiana. ¿Cuál es
el procedimiento actual que utilizaría para confirmar el diagnóstico?

A coprocultivo

B detección de anticuerpos

C serologia

D Trofozoitos en heces

Mujer de 58 años, hospitalizada de urgencia con diagnóstico de
enfermedad vascular cerebral. ¿cuál de las siguientes opciones
corresponde al tipo de accidente vascular mayormente relacionado
con riesgo de muerte en la paciente?

A isquémico

B trombótico

C embólico

D hemorrágico

Femenina de 35 años, hospitalizada con diagnóstico de pancreatitis
aguda. ¿Cuál es el factor de riesgo más importante en la patogenia de
esta enfermedad?

A género

B edad

C colecistitis

D obesidad

Hombre de 33 años de edad que ingresa con cuadro compatible de
síndrome de stevens johnson. A la exploración se encuentra 30% de
superficie corporal cutánea afectada. Con base en la superficie
afectada, ¿Cómo podemos clasificar la enfermedad?

A Síndrome de stevens johnson

B Síndrome de stevens johnson leve

C Superposición síndrome de stevens johnson-necrólisis epidérmica
tóxica

D Necrólisis epidérmica tóxica

La probabilidad de morir en este paciente se incrementa si no es
referido a una unidad especializada en el manejo de éste tipo de
pacientes antes de:

A Los 4 primeros días de la enfermedad

B Los 5 primeros días de la enfermedad

C Los 6 primeros días de la enfermedad

D Los 7 primeros días de la enfermedad

Hombre de 48 años con diagnóstico reciente de enfermedad de
parkinson. Tratado con levodopa que al transformarse en dopamina
en el sistema nervioso central se espera compense la lesión de la
siguiente área:

A sustancia negra mesencefálica

B bulbo raquídeo

C sustancia gris del prosencéfalo

D núcleo arqueado del hipotálamo

Mujer de 23 años de edad es ingresada al servicio de urgencias con
diagnóstico de intoxicación por cocaína. ¿Cuál es el tratamiento
inicial?

A diacepam

B fenelzina

C difenilhidantoína

D haloperidol

Durante su rotación por el servicio de oftalmología ha notado un
incremento considerable en los pacientes con deterioro en la visión.
¿Cuál es la causa más probable en la mayoría de los casos?

A defectos congénitos

B traumatismos

C glaucoma

D diabetes

Durante un estudio retrospectivo en el hospital donde se encuentra
realizando sus prácticas se ha visto un notable incremento en la
incidencia de insuficiencia renal crónica por lo que es necesario
conocer la causa principal de esta enfermedad con la finalidad de
actuar tempranamente en su prevencíón y diagnóstico. ¿Qué es lo
más probable como primera causa de esta enfermedad?

A Nefropatía hipertensiva

B Malformaciones congénitas

C Nefropatía diabética


Mujer de 20 años con diagnóstico actual de anorexia y raquitismo
secundario. En esta paciente deberá indicarse que se exponga al sol
debido a que en la piel se lleva a cabo parcialmente el metabolismo
del:

A calciferol

B retinol

C tocoferol

D ascorbato

Hombre de 51 años con diagnóstico de cirrosis, actualmente con
sangrado de tubo digestivo alto por varices esofágicas, refractario a
tratamiento. ¿Cuál es el tratamiento más efectivo para controlar este
tipo de sangrado?

A Colocación de sonda de balones

B Administración de propanolol

C Administración de somatostatina

D Realización de derivaciones quirúrgicas

Mujer de 68 años de edad que es evaluado en el servicio de urgencias
por caída en su domicilio hace 3 horas, con pérdida del estado de
alerta por 10 minutos. Durante el interrogatorio se corrobora con
familiar la administración de 15 médicamentos diarios. Se considera
que la ingestión de medicamentos es causa del siguiente porcentaje
de caídas en el adulto mayor:

A 1%.

B 5%.

C 10%.


Por frecuencia es más probable que el lugar en la casa en la que sufrio
la caída la paciente es:

A patio

B baño

C escalera

D recámara

¿Cuál es el mecanismo de la caída más frecuentemente asociado a
lesiones en el paciente adulto mayor?

A el tropiezo

B el resbalón

C la pérdida del equilibrio

D lo empujaron

Mujer de 19 años de edad. Durante una campaña de salud realizada
recientemente resulto positiva para la detección de sobrepeso y
obesidad. Niega enfermedades crónicas en familiares de primero y
segundo grado. Por antropometría se obtuvo un imc de 32.6. ¿Qué
tipo de obesidad tiene la paciente?

A sobrepeso

B obesidad I

C obesidad II

D obesidad clase III

Corresponde al déficit calórico más recomendado que se deberá
aplicar a la dieta actual de la paciente:

A 200 a 500 kcal/día

B 500 a 1000 kcal/día

C 1000 a 1200 kcal/día

D 1200 a 1500kcal/día

Mujer de 50 años vendedora ambulante en una playa desde hace 25
años. Acude a su consulta al notar una lesión en mejilla derecha que
no ha desaparecido desde hace 3 meses. A la exploración encuentra
lesión ovoide de 2 cm de diámetro, con bordes irregulares, de color
rojo vino. ¿Cuál es el tratamiento de elección para este paciente?

A radioterapia

B quimioterapia

C resección completa de la lesión

D electrofulguración

Hombre de 76 años de edad, que está siendo valorado en la consulta
de primer nivel por probable síndrome demencial. Usted considera la
posibilidad de que este síndrome sea secundario a alteración orgánica
estructural. Con base en esta posibilidad, lo indicado será:

A Iniciar de inmediato manejo farmacológico con antidepresivos

B Solicitar tac de cráneo

C Derivar a segundo nivel de atención

D Envío inmediato a cámara hiperbárica

Durante su año de servicio social se le indica una meta de cobertura
total de habitantes mayores de 60 años con vacuna de influenza
estacional. Para este fin ha tenido a bien realizar pláticas
educacionales de promoción a la salud y visitas domiciliarias en toda
la comunidad. Las razones más frecuentes por las cuales los pacientes
rechazan la vacunación son por la asociación a efectos adversos y la
duda en cuanto a su efectividad. Usted deberá informar a los
pacientes que, el siguiente es un factor asociado al huésped que
disminuye la efectividad de la vacuna:

A Edad

B Presencia de enfermedad respiratoria aguda

C Antecedente de infección por influenza

Mujer de 25 años con diagnóstico reciente de esclerosis múltiple con
interferón beta. La justificación para su indicación se basa en la
siguiente propiedad inmunomoduladora:

A Inhibición de las células presentadoras de antígeno

B Inhibición de las citocinas reguladoras

C Aumento en la proliferación de linfocitos t

D Limitación del movimiento de células inflamatorias en el SNC

Mujer de 25 años con diagnóstico reciente de cárcinoma papilar de
tiroides en estadio I. ¿Cuál es el pronóstico de curación a 20 años en
este caso?

A Arriba del 95%

B Entre 90 y 94%

C Entre 85 y 89%

D Entre 80 y 84%

Hombre de 25 años, alcohólico desde hace 10 años, toma de 1 a 2
litros de alcohol diario hasta llegar a la embriaguez. A la exploración
se encuentra con desnutrición severa, adecuada coloración en piel,
cor anémico y datos de hepatítis alcohólica severa. el motivo por el
cual clínicamente no es posible apreciar ictericia en el paciente es
debido a que la principal fuente de bilirrubina es a partir de:

A los aminoácidos de la mioglobina

B la beta globina de la hemoglobina

C el grupo hem de la hemoglobina


Hombre de 58 años, fumador desde hace 25 años, actualmente con
diagnóstico de EPOC. ¿Cuál es la intervención más eficaz para evitar
complicaciones en pacientes con este diagnóstico?

A Administración de antibióticos profilácticos

B Administración de broncodilatadores permanentes

C Vacunación contra la influenza

D Suministro de oxígeno permanente

Hombre de 55 años con antecedente de diabetes mellitus tipo 2 de 18
años de evolución. Con diagnóstico reciente de insuficiencia renal
crónica terminal. ¿Cuál es una indicación absoluta para iniciar diálisis
en el paciente?

A Acidosis con un ph de 7.3

B Sobrecarga de volumen refractaria.

C Depuración de creatinina menor a 40 ml/min.

D Creatinina sérica mayor a 3 mg/dl.+J59

Hombre de 36 años de edad, extranjero quien desde hace 6 meses
llega a radicar en una comunidad rural de nuestro país. Presenta
desde hace 1 semana fiebre mayor a 38°c para lo cual se automedica
paracetamol, hace 2 días se agrega tinte ictérico y dolor intenso en
cuadrante superior derecho. A la exploración se encuentra paciente
febril, conjuntivas y piel ictérica, abdomen distendido, peristalsis
aumentada, hepatomegalia dolorosa. Se solicita ultrasonido el cual
reporta absceso hepático en lóbulo derecho único. Las pruebas de
laboratorio sugieren origen amebiano. ¿Cuál es el tratamiento de
elección en este momento?

A nitazoxanida

B metronidazol

C drenaje del abceso

D tinidazol

Mujer de 22 años de edad que acude al servicio de urgencias, cuenta
con el antecedente de hace un mes de haber presentado intento
suicida de alta letalidad. Sus padres la refieren aislada, sin amigos, con
rendimiento académico bueno hasta hace 8 meses en que abandonó
la escuela aduciendo que sus compañeros controlaban sus
pensamientos y la obligaban a autolesionarse. A la exploración se
encuentra inquieta, desesperada, muestra soliloquios y suspicacia. No
establece contacto visual con el médico entrevistador. ¿Cuál es el
diagnóstico clínico más probable?

A personalidad límite

B autismo

C esquizofrenia

D trastorno de ideas delirantes

¿Cuál es la conducta médica que deberá establecerse como medida
terapéutica en este caso?

A antipsicóticos atípicos

B terapia cognitivo-conductual

C benzodiazepinas

D estabilizadores del ánimo

Hombre de 43 años de edad, con hipertensión arterial e insuficiencia
renal crónica estadio I. Se ha considerado que en este tipo de
pacientes el factor que condiciona la progresión de la enfermedad son
las fluctuaciones constantes en la presión de la corriente sanguínea
que llega hacia el riñón. ¿Cuál es la primera línea de defensa contra
este tipo de fluctuaciones?

A Angiotensina II

B Retroalimentación tubulo glomerular

C Reflejo miógeno


Mujer de 23 años, con diagnóstico reciente de rinitis alérgica. Durante
su valoración usted deberá saber que esta enfermedad es mediada
por la siguiente inmunoglobulina:

A A

B G

C M

D E

Hombre de 73 años, desnutrido, con diagnóstico reciente de anemia
por deficiencia de hierro. Usted indicará hierro por vía oral como
parte del tratamiento y lo complementará con alimento ricos en
hierro. ¿Cuál es la medición que más rápidamente le indicará si el
paciente esta respondiendo al tratamiento con hierro?

A Cuenta de reticulocitos

B Hemoglobina

C Hemoglobina corpuscular media

D Volumen corpuscular medio

Hombre de 40 años de edad, que ingresa con diagnóstico de
síndrome anémico. A la exploración se encuentra ictericia en
escleróticas, palidez leve de tegumentos y esplenomegalia. Se
reportan exámenes de laboratorio con hemoglobina de 8 g/dl,
leucocitos 9000, plaquetas 275 000, reticulocitos 8.8%. El frotis de
sangre periférico corrobora la presencia de reticulocitos y la presencia
de esferocitos. ¿Cuál es el tratamiento indicado en este momento en
el paciente?

A Esplenectomia

B Danazol

C Transfusión de concentrado eritrocitario

D Prednisona

Mujer de 45 años, fumadora de 20 años, actualmente en estudio por
aparente neumopatía cronica. Para hacer diagnóstico diferencial entre
asma y epoc el dato fundamental de la espirometría que nos
confirmará el diagnóstico de asma es:

A Patrón obstructivo no reversible

B Patrón restrictivo no reversible

C Patrón obstructivo reversible

D Patrón restrictivo reversible

Paciente femenino de 38 años de edad, vih positivo desde hace 10
años y diagnóstico de sida, desde hace 6 meses. Cursa con
diagnóstico reciente de encefalitis por toxoplasma. ¿Cuál es el
tratamiento de primera elección en este caso?

A pirimetamina, sulfadiazina y ácido folínico

B albendazol y con nitozoxamida

C anfotericina b, flucitosina y fluconazol D mefloquina en
combinación con ofloxacina

Mujer de 30 años, diagnosticada con constipación crónica no
complicada desde hace 6 meses. El manejo ha consistido
exclusivamente en medidas higienico dietéticas, sin embargo la
paciente persiste con el malestar. ¿Cuál es la siguiente medida
terapéutica a seguir?

A indicar bloqueadores de los canales de calcio

B realizar enemas evacuantes jabonosos

C indicar procinético

D indicar psyllium plantago

Mujer de 45 años, habita en medio rural, cocina con leña desde los 10
años. Acude con sintomatología respiratoria de 2 años de evolución.
El siguiente síntoma o signo clínico es considerado el más importante
para considerar el diagnóstico de enfermedad pulmonar obstructiva
crónica:

A tos crónica

B producción regular de esputo

C sibilancias

D disnea

Adulto de 27 años, tabaquismo positivo desde los 17 años de edad,
consume al menos 10 cigarrillos al día. Acude a consulta por presencia
de tos seca en accesos recurrente. Se le indica la necesidad de
abandonar el hábito tabáquico a lo cual responde positivamente pero
solicita un período mayor a dos meses para abandonar su consumo
por completo. ¿En que fase se encuentra para el abandono del hábito
tabáquico?

A fase de pre-contemplación

B fase de contemplación

C fase de preparación

Femenina de 79 años previamente sana. Acude a su consultorio por
presencia de dolor matutino en manos de mediana intensidad. En la
exploración presenta dolor en la muñeca derecha, presencia de
nodulaciones interfalángicas distales y proximales de forma bilateral.
A la digitopresión refiere dolor también en cadera y rodillas. Las
radiografías de manos muestran estrechamiento del espacio articular.
¿Cuál es el diagnóstico clínico mas probable?

A Artritis infecciosa

B Artritis reactiva

C Artrosis

D Artritis reumatoide

Mujer de 36 años, con antecedente de artritis reumatoide
actualmente en remisión completa desde hace 3 meses. Hace 1
semana suspende metrotexate y hasta el momento se encuentra
asintomática. ¿Cuál es la conducta a seguir más adecuada en esta
paciente?

A Reiniciar tratamiento con antinflamatorios no esteroideos

B Iniciar en este momento terapia física

C Reiniciar medicamentos modificadores de la enfermedad

D Mantener en vigilancia y dar datos de alarma
crónico. Refiere desde hace una semana hipertermia cuantificada por
arriba de 38.5°c de predominio vespertino, dolor abdominal en flanco
derecho y diarrea sólo en 1 o 2 ocasiones. A la exploración física
temperatura de 39°c, disnéico, ictericia generalizada ++, abdomen
globoso con hepatomegalia dolorosa, con borde a 5cm por debajo del
reborde costal. Los estudios de laboratorio revelan anemia
normocítica normocrómica, leucocitosis, enzimas hepáticas elevadas
y elevación de la fosfatasa alcalina. La radiografía de tórax muestra
elevación del hemidiafragma derecho. Como parte del abordaje del
paciente, ¿qué debemos solicitar en este momento?

A Tomografía axial computada

B Radiografía de abdomen

C Ultrasonido hepático

D Panel viral de hepatitis

Mujer de 28 años de edad presenta desde hace 9 meses dermatosis
pruriginosa diseminada en la piel cabelluda, pliegues nasogenianos,
alas nasales, mejillas y frente, caracterizada por placas eritematosas
con descamación fina, adherente y oleosa. ¿Cuál es el diagnóstico
clínico más probable?

A dermatitis atópica

B psoriasis

C dermatitis por contacto

D dermatitis seborreica

Hombre de 33 años con diagnóstico de gota. Actualmente en
tratamiento con alopurinol, que se justifica debido a que:

A favorece la degradación hepática de purinas

B incremento en la modificación de la actividad de la fosforribosil
transferasa

C favorece la degradación acelerada de nucleótidos

D inhibe a la xantina oxidasa

Mujer de 43 años de edad, diabética acude a consulta refiriendo que
desde hace 3 semanas presenta manchas en diversas partes del
cuerpo. A la exploración física: obesidad ii, máculas hipocrómicas de
tonalidad rosada, con diámetro menor a 1cm, múltiples, en cara
posterior de cuello, tórax y axilas, las cuales muestran escasa
descamación al rasparlas. ¿Cuál es el diagnóstico más probable en
esta paciente?

A pitiriasis versicolor

B lupus eritematoso cutáneo

C vitiligo

D micosis fungoide

¿cuál es el tratamiento de elección en este caso?

A champú de flutimazol

B sulfato de cloroquina

C fototerapia ultravioleta

D esteroide tópico

Hombre de 43 años de edad, actualmente en su cuarta semana de
tratamiento acortado estrictamente supervisado para tuberculosis
pulmonar. Los estudios generales de control realizados el día de ayer
reportan elisa positiva para vih, misma que es confirmada mediante
inmunofluorescencia. Se decide iniciar con esquema antirretroviral
específico Es una complicación que puede presentarse en este
paciente una vez que inicie el manejo antiretroviral:

A síndrome de reconstrucción inmune

B leucoencefalopatía multifocal progresiva

C neumonía atípica

D angiomatosis bacilar

Mujer de 43 años de edad, quien presenta desde hace 2 meses
lesiones tipo ronchas pruriginosas en todo el cuerpo que aparecen y
desaparecen durante el día de forma espontánea. Se le han indicado
diversos tratamientos tópicos sin mejoría clínica. En este momento sin
lesiones. Por el antecedente se clasifica como una urticaria crónica y
se indican medicamentos con acción en receptores h1 y h2 de la
histamina. Por frecuencia lo más probable es que la paciente presente
una urticaria:n

A autoinmune

B física

C idiopática

D alérgica

¿Cuál es es el medicamento que cumple las características de inhibir
los receptores h1 y h2 de la histamina?

A difenhidramina

B doxepina

C cimetidina

D desloratadina

Mujer de 60 años de edad, acude a consulta durante un brote de
amibiasis intestinal en su comunidad, presentando distensión
abdominal, meteorismo, pujo, tenesmo y evacuaciones con sangre
fresca. ¿Cuál es el mecanismo de acción del antibiótico indicado en
este paciente?

A inmovilizar a los trofozoitos a nivel intraluminal evitando así su
propagación

B provoca una reacción química reductiva intracelular que lesiona los
ácidos nucléicos bacterianos

C evita la incorporación de nuevos aminoácidos a la cadena peptídica
en crecimiento

D actúa inhibiendo la síntesis de arn mensajero, a cargo de la arn
polimerasa

Mujer de 53 años que durante su estudio por un síndrome anémico
de 6 meses de evolución, se identifica disminución de síntesis de
hemoglobina. ¿A qué se debe este hallazgo fisiopatológico?

A Deficiencia de glucosa 6-fosfato deshidrogenasa

B Deficiencia de folatos

C Deficiencia de vitamina b12

D Deficiencia de hierro

Mujer de 66 años de edad con derechohabiencia, es enviada a
segundo nivel con el diagnóstico de incontinencia urinaria. ¿Qué
porcentaje de los pacientes con este diagnóstico demanda atención
médica en México?

A más del 80%

B más del 70%

C menos del 70%

D menos del 50%

Masculino de 40 años con antecedente de cardiopatía isquémica
acude a valoración nutricional para disminuir factores de riesgo. ¿Cuál
es el sitio de depósito de tejido adiposo que confiere un mayor riesgo
de morbilidad en éste paciente?

A muscular

B subcutáneo

C abdominal

D visceral

Mujer de 36 años de edad con obesidad clase 2, desde hace 3 meses
en tratamiento nutricional y actividad física programada. Actualmente
con mejoría en cifras tensionales, lípidos y glicemia sérica, pero sin
lograr la reducción de peso esperada. Se decide iniciar tratamiento
farmacológico para la reducción de peso con orlistat. ¿Cuál es el
mecanismo de acción del medicamento indicado en esta paciente?

A inhibidor en la recaptura de serotonina

B inhibidor de la glucosidasa alfa

C antagonista selectivo del receptor cannabinoide

D inhibidor de la lipasa intestinal
refiriendo desde hace un año nota rigidez de piel, dolor articular de
predominio en manos y rodillas, dolor abdominal de predominio en
epigastrio y malestar general. A la exploración presenta
engrosamiento de piel de predominio en cara, edema de manos y
artritis en muñeca derecha y rodilla del mismo lado. Presenta además
fenómeno de raynaud. Trae resultados de laboratorio que reporta
anticuerpos antinucleares positivos. ¿Cuál es el diagnóstico más
probable en este paciente?

A vasculitis reactiva

B esclerosis sistémica

C lupus eritematoso generalizado

D artritis reumatoide.

Hombre de 60 años de edad con antecedente de hipertensión y
cardiopatía mitral. Ingresa a urgencias por hemiparesia facio corporal
izquierda de inicio súbito. La exploración presenta ta 140/85 mm/hg,
fc 98 x`, fr 23 x`, ruidos cardíacos arrítmicos con soplo mitral, sin datos
de congestión pulmonar. Se corrobora hemiparesia faciocorporal
izquierda. ¿cuál es el diagnóstico clínico más probable en esta
paciente?

A isquemia cerebral transitoria

B emergencia hipertensiva

C embolia cerebral

D hemorragia cerebral

¿Cuál es el tratamiento de mayor eficacia para disminuir el riesgo
relativo de otro evento en esta paciente?

A enalapril

B aspirina

C clopidogrel

D warfarina

Masculino de 71 años de edad, presenta desde hace 7 semanas fiebre
y diaforesis nocturna. En los últimos días se agrego disnea, que ha ido
progresando de grandes a pequeños esfuerzos. En la exploración
física se encuentra soplo sistólico de predominio mitral, sin
irradiaciones, edema en extremidades inferiores y lesiones nodulares
en pulpejos de dedos de ambas manos. ¿Cuál es el diagnóstico clínico
mas probable?

A endocarditis postoperatoria tardía

B endocarditis fulminante

C endocarditis aguda

D endocarditis subaguda

Hombre de 32 años, taxista, que desde hace 15 días no puede
trabajar al presentar nerviosismo extremo, palpitaciones, disnea,
calambres, diarrea, fiebre y pérdida de peso no cuantificada. A la
exploración: ta 120/75, fc 130/min, fr 26/min, ruidos cardiacos
arrítmicos, edema leve de miembros inferiores. ¿Cuál es el
diagnóstico más probable?

A hiperparatiroidismo primario

B feocromocitoma

C tirotoxicosis

D síndrome carcinoide

Hombre de 60 años que acude a consulta con resultado de tasa de
filtración glomerular de 35 ml/min/1.73m2. Con base en este
resultado su plan de acción terapéutico consistirá en:

A reducción de factores de riesgo para enfermedad renal crónica

B manejo nefroprotector y tratamiento de complicaciones

C preparar para terapia de reemplazo renal

D comenzar programa de diálisis

Paciente masculino de 45 años con diagnóstico reciente de
enfermedad de parkinson, tratado actualmente con levodopa.
Agregar carbidopa al tratamiento de este paciente es para obtener el
siguiente beneficio:

A favorecer la absorción oral de la levodopa

B aumentar la afinidad de la levodopa por el receptor D2

C inhibir el metabolismo hepático de la levodopa

D inhibir la dopa descarboxilasa periférica

Hombre de 68 años de edad, con antecedente de angina de pecho
estable y diabetes; en tratamiento con nifedipino y metformina. Es
llevado a consulta ya que desde hace una semana presenta en tres
ocasiones desvanecimiento con pérdida transitoria del estado de
alerta. En este momento asintomático. Lo más probable es que el
paciente haya presentado:

A Síncope neurogénico

B Hipotensión ortostática

C Síncope cardiogénico


¿Qué debe de indicarse como parte del manejo del paciente?

A Ácido valpróico

B Suspender vasodilatador

C Pseudoefedrina

D Implante de marcapaso definitivo

Hombre de 28 años de edad que ingresa con lesiones cutáneas
compatibles con síndrome de stevens johnson. A la exploración se
encuentra entre 30 y 40% de superficie corporal cutánea afectada.
Con base en la superficie afectada, ¿cómo podemos clasificar la
enfermedad?

A Síndrome de stevens johnson

B Síndrome de stevens johnson leve

C Superposición síndrome de stevens johnson-necrólisis epidérmica
tóxica.


El pronóstico de la paciente será mejor si es ingresada a:

A Medicina interna

B Tercer nivel de atención

C Unidad de cuidados intensivos

D Unidad de atención a quemados

Hombre de 32 años de edad, tabaquismo positivo intenso, acude a
consulta solicitando atención por presencia de diarrea; con esta se
registran al menos 5 episodios similares en 6 meses, mismos que se
acompañan de dolor abdominal persistente que alterna con días o
semanas en las cuales hay evacuaciones normales. A la exploración
física temp 38.5°c, mucosa oral hidratada, aftas peribucales, abdomen
globoso distendido doloroso en fosa ilíaca derecha, se realiza
exploración rectal encontrándose fisuras perianales y absceso
perianal de no más de 2cm de diámetro. La biometría hemática
reporta leucocitosis, aumentos de la vsg y anemia normocítica
hipocrómica. La radiografía de abdomen con contraste muestra
aspecto en empedrado en las asas intestinales. ¿Cuál es el diagnóstico
clínico más probable en este paciente?

A Enfermedad de crohn

B Síndrome de colon irritable

¿Cuál es el tratamiento de elección en este caso

A Dieta con alto contenido de fibra y amoxicilina

B Quirúrgico más radioterapia

C Antiespasmódicos + psicotrópicos

D Alimentación baja en grasas + metronidazol

Hombre de 19 años que en la calle presenta paro cardio respiratorio
súbito. usted comienza a asistirlo aplicándole reanimación
cardiopulmonar básica cuyo objetivo principal es:

A obtener una vía áerea permeable

B lograr la ventilación pulmonar

C mantener el gasto cardíaco y los pulsos

D mantener la perfusión de órganos

Mujer de 33 años que acude a la consulta por presentar desde hace 3
meses prurito nasal, estornudos en salva y moco nasal. Para
completar el cuadro clínico de rinitis alérgica el horario de aparición
de los síntomas deberá ser principalmente durante:

A La noche

B La madrugada

C La tarde

D La mañana

Mujer de 84 años de edad, obesa, con fractura de tibia lo cual le
impide la deambulación desde hace 3 semanas. Solicita atención
médica por presencia de herida en región glútea derecha. A la
exploración se observa lesión de aproximadamente 9cm de diámetro
conformada por una vesícula central con bordes eritematosos,
dolorosa a la palpación. ¿Cuál es el estadio de la úlcera presente en la
paciente?

A Estadio I

B Estadio II

C Estadio III

D Estadio IV

¿Cuál es el manejo inicial de la lesión?

A Aspirado aséptico de la flictena

B Antibióticos tópicos

C Debridar la epidermis de la flictena

D Antisépticos tópicos

Femenina de 35 años de edad, con antecedente de diarrea acuosa de
2 meses de evolución. Durante su estudio se diagnóstica intolerancia
a la lactosa. ¿cuál es el mecanismo fisiopatológico que produce la
diarrea es debido a este disacárido?

A estimula las vellosidades

B se intercambia por otros azúcares

C no se hidroliza y es osmótico

D es utilizado por la flora normal

Mujer de 42 años de edad, internada desde hace 10 días por
debilidad progresiva, simétrica y ascendente de miembros inferiores e
hiporreflexia, sin fiebre asociada. Refiere cuadro gripal 15 días antes
del inicio de los síntomas. El día de hoy se tomará nueva muestra de
líquido cefalorraquídeo, ya que la de su ingreso fue normal. ¿cuál es el
hallazgo más probable en esta nueva muestra de líquido
cefalorraquídeo?

A pleocitosis

B proteinorraquia

C glucorraquia

D presión de inicio elevada

Hombre de 52 años con diagnóstico reciente de artritis reumatoide.
Decide comenzar manejo con el medicamento cuyo mecanismo de
acción consiste en inhibir la vía de los folatos:

A sales de oro

B tamoxifeno

C metrotexato

D colchicina

Hombre que acude a su consulta con diagnóstico de migraña desde
hace 15 años. Actualmente sin medicamento para profilaxis. Acude a
su consulta con aparente nuevo evento de migraña muy intenso.
Usted decidirá enviarlo a urgencias de su hospital en caso de:

A Edad mayor de 55 años

B Inicio insidioso

C Duración mayor a 12 horas del evento

D Incremento de la intensidad con estímulos auditivos y visuales

Mujer de 33 años de edad, adicta a la cocaína inhalada y a la heroína
intravenosa. Acude a consulta al presentar cefalea, fiebre no
cuantificada, mialgias y adenomegalias. Considera usted, la
posibilidad de cursar con un síndrome retroviral agudo. Para
considerar este síndrome; hace cuanto tiempo como mínimo, tendría
que haber adquirido la infección por vih esta paciente:

A mínimo hace 9 semanas

B mínimo hace 8 semanas

C mínimo hace 6 semanas


Mujer de 69 años de edad, en cama desde hace varios meses como
consecuencia de una resección de tumor cerebral. Desde hace 1 mes
presenta deterioro progresivo de su estado general, hipertermia no
cuantificada y tos productiva. En la exploración física se encuentra
temperatura 38 °c, taquicárdica, taquipnéica, con cianosis distal,
higiene dental deficiente y halitosis. Reflejo nauseoso ausente, se
ausculta soplo tubárico y anfórico en hemitórax derecho. ¿Cuál es
diagnóstico clínico más probable?

A bronconeumonía y paquipleuritis

B bronquiectasias sobreinfectadas

C neumonía grave adquirida en la comunidad

D absceso pulmonar

¿Cuále es el tratamiento de elección para esta paciente?

A practicar resección quirúrgica

B administrar ceftriaxona más metronidazol

C administrar claritromicina

D administrar tetraciclina

Mujer de 18 años de edad presenta desde hace 3 meses candidiasis
vaginal recurrente, polidipsia y pérdida de peso de 4kg. A la
exploración índice de masa corporal debajo del percentil 1 para la
edad. Se sospecha diabetes mellitus tipo 1 por lo que se solicita en
este momento una determinación de glucemia sérica. ¿Cuáles son lo
niveles de glucosa que confirman el dx?

A mayor a 125mg/dl

B mayor a 150mg/dl

C mayor a 200mg/dl


¿Cómo debe iniciarse el tratamiento?

A Insulina basal y en bolos

B Dos inyecciones al día de insulina premezclada

C Una dosis de insulina

D Insulinas nph 2 veces al día

Hombre de 38 años de edad, homosexual, vih positivo. Desde hace 3
meses presenta fiebre intermitente, astenia, hiporexia, diaforesis
nocturna, pérdida de peso no cuantificada y adenopatía en cuello. A
la exploración se encuentra adenopatía de aproximadamente 3 cm en
la cara lateral izquierda de cuello, fija a planos profundos, no dolorosa
y bordes regulares. En abdomen se encuentra esplenomegalia. ¿Cuál
es el diagnóstico clínico más probable?

A Leucemia mieloide crónica

B Linfoma no hodgkin

C Síndrome de inumonodeficiencia adquirida

D Tuberculosis ganglionar

¿Cuál es el tratamiento del paciente?

A trasplante de médula ósea

B quimioterapia

C antirretrovirales

D antifímicos

Para realizar el diagnóstico definitivo usted deberá indicar el siguiente
procedimiento:

A Aspirado de médula ósea

B Biopsia de la adenopatía

C Serología para VIH

D Reacción en cadena de ligasa

Hombre de 70 años de edad, con antecedente de diabetes mellitus
tipo 2 e hipertensión arterial desde hace 20 años. Es encontrado, en
el baño tirado con laceración en la región temporal, disartria y cefalea
generalizada. A su llegada a urgencias a la exploración se encuentra ta
160/105 mmhg, f.c: 85 x´, f.r: 25 x´ y glasgow entre 13 y 14. Se
reportan exámenes de laboratorio con creatinina 1.5 mg/dl, bun 25,
glucosa 180 mg/dl. Se indica tac simple y contrastada de urgencia que
reporta hematoma subdural en región temporal. A las 48 hrs de su
hospitalización presenta fiebre y tos con expectoración amarillo-
verdosa. Creatinina 3 mg/dl, bun 35, fena 2.6 %, sodio urinario 45
mmol/l, glucosa 150 mg/dl, bh: leucocitos 13, 000, neutrófilos 60%,
eosinofilos 15%. El siguiente evento fisiopatológico explica la
insuficiencia renal aguda presente en el paciente:

A Toxicidad epitelial parenquimatosa renal e infiltración eosinofílica B
Isquemia intersticial renal e isquemia medular renal

C Isquemia medular renal y toxicidad tubular renal

¿cómo podemos confirmar el diagnóstico?

A estudio del sedimento urinario

B examen general de orina

C ultrasonido renal

D depuración de creatinina de 24 hrs

La insuficiencia renal aguda se hubiera prevenido a su ingreso
mediante la indicación de:

A n-acetilcisteína

B uso de contraste iónico

C bolo de 100 mgs i.v. de hidrocortisona

D hidratación con solución salina 0.9 %

Femenina de 38 años hospitalizada en el servicio de medicina interna.
Se sospecha la posibilidad de cursar con insuficiencia suprarrenal
secundaria. Solicita niveles hormonales, para confirmar el diagnostico
los resultados serian:

A ACTH baja y cortisol sérico bajo

B ACTH elevada y cortisol sérico elevado

C ACTH alta y cortisol sérico bajo

D ACTH baja y cortisol sérico elevado

Mujer de 45 años abogada divorciada y madre de tres hijos. Desde
hace un año, después de la muerte de su hermana se ha sentido triste
e irritable. Prácticamente todos los días desde hace 6 meses no siente
deseo de arreglarse. Trabaja más por obligación que por gusto y su
desempeño es calificado como deficiente. Duerme mucho y no
descansa. Se siente lenta y torpe. En el último mes, ha pensado que
sería mejor morir a seguir viviendo así. ¿Cuál es el diagnóstico más
probable?

A Depresión mayor

B Duelo con síntomas depresivos


Corresponde al incremento del riesgo de suicidio de la paciente:


A Menos del 5%

B Del 10 al 15%

C Del 20 al 25%

D Más del 25%

¿Cuál es el tratamiento indicado?

A Farmacoterapia antidepresiva

B Terapia electroconvulsiva

C Psicoterapia analítica

D Terapia cognitivo conductual

Hombre de 25 años de edad, con diagnóstico de vih que acude a
control a su consulta. Recibiendo tratamiento con antirretrovirales
con adecuado apego. Para considerar al paciente portador de una
infección crónica por VIH, debería cumplir con al menos:

A 1 año desde la infección

B 3 años desde la infección

C 5 años desde la infección

D 7 años desde la infección

Masculino de 23 años, con diagnóstico reciente de anemia de células
falciformes. Este paciente presentará mejoría si es tratado con:

A ácido fólico e hidroxiurea

B cobalamina

C hierro y eritropoyetina

D prednisona y esplenectomía

Paciente masculino de 65 años con diagnóstico de síndrome anémico
y que comienza estudio por probable anemia secundaria a deficiencia
de hierro. La biometría hemática sugiere este diagnóstico. Se espera
se eleve con rapidez tras el tratamiento aún cuando los depósitos de
hierro no estén repuestos por completo:

A La saturación de transferina sérica

B Los niveles de ferritina sérica

C El volumen corpuscular medio

D La capacidad total de fijación al hierro

Hombre de 55 años de edad, neumópata crónico con diagnóstico de
neumopatía intersticial difusa. La prescripción de oxígeno
suplementario en este paciente debe indicarse:

A si la presión de dióxido de carbono sea mayor a 45 mmhg.

B si existen comorbilidades cardíacas asociadas.

C si aparece cianosis y disnea de medianos esfuerzos.

D si la presión arterial de oxígeno llega a 55 mmhg ó menos.

Mujer de 56 años en estudio actualmente por probable anemia
ferropriva. Acude con resultados de laboratorio donde usted esperaría
encontrar el siguiente hallazgo para confirmar su sospecha
diagnóstica:

A Elevación del receptor de transferrina

B Hierro sérico elevado

C Ferritina elevada

D Aumento de la saturación de transferrina

Hombre de 32 años, con antecedentes familiares de urolitiasis. Usted
deberá señalarle que el siguiente factor de riesgo se asocia
fuertemente con el desarrollo de urolitiasis:

A Ingesta baja de agua

B Hipocalciuria

C Consumo bajo de proteínas animales

D Ingesta alta de magnesio

Mujer de 60 años de edad. Hace 3 años padeció pancreatitis aguda
biliar grave y necrosis tubular aguda renal, sin seguimiento ni control
posterior. Cuadro clínico progresivo de 7 días de evolución,
caracterizado por anorexia, náuseas, vómito ocasional, prurito
generalizado y dolor abdominal epigástrico, además de dolor torácico
a la inspiración profunda. A la exploración se encuentra ta 160/100
mm/hg, f.c: 115 x', f.r: 26 x', temp 36ºc, asterixis y somnolencia. Los
laboratorios reportan hb 10.1, leucocitos 10 000, plaquetas 155,000,
creatinina 8.0, bun 150 mg/dl, sodio 155, k 7, cl 115 gasometría: ph
7.23, hco3 11, co2 21 , sat o2 89 %. ¿Cuál es el diagnóstico clínico más
probable?

A Choque séptico compensado

B Insuficiencia renal aguda

C Síndrome urémico

¿Cuál es el tratamiento que debe de establecerse de inmediato

A carga de 500 ml de solución salina isotónica, diuréticos y
salbutamol

B transfusión de sangre y diálisis peritoneal

C bicarbonato de sodio y soluciones polarizantes

D gluconato de calcio y hemodiálisis

Mujer de 58 años de edad, postoperada de safenectomía. Ingresa a la
terapia intensiva con sospecha de tromboembolia pulmonar. El
estudio que tendrá la mayor sensibilidad para confirmar el
diagnóstico la paciente es la determinación de:

A Monomeros

B Dímero D

C Fibrinogeno sérico

D Transaminasa glutámico oxalacética

Mujer de 58 años con antecedente de hipertensión arterial de 25
años de evolución. Desde hace 2 semanas presenta disnea de
pequeños esfuerzos. A la exploración presenta fc 120/min, ta 90/40
mmhg, fr 28/min, desorientación, piel pálida y fría, estertores
crepitantes en ambos hemitórax, edema de miembros inferiores hasta
cadera. Ecg: ritmo sinusal, eje desviado a la izquierda. El paciente
presenta insuficiencia cardíaca:

A De gasto alto

B Global

C Derecha

Mujer de 35 años de edad, acude a consulta por presentar desde hace
8 días fiebre de hasta 39 grados. Inició con odinofagia, tos y
expectoración. A las 48 horas presentó un exantema localizado en
tronco, el cual desapareció en un día. El día de hoy continúa con
fiebre vespertina, diarrea y mayor ataque al estado general. En la
exploración se encuentra hepatomegalia y dolor a la palpación del
marco cólico. ¿Cuál es el estudio de mayor utilidad para confirmar el
diagnóstico etiológico?

A La serología

B La biometría hemática


Mujer de 52 años, acude a su consultorio por presentar desde hace 5
meses dolor de predominio matutino e inflamación articular en
manos, codos y rodillas, mejorando conforme transcurre el día. A la
exploración se encuentra con palidez, artritis de articulaciones
metacarpo falángicas de forma bilateral, deformación en cuello de
cisne. En codo izquierdo se encuentra nodulación no dolorosa.
Presenta artritis de rodilla izquierda. Se reporta factor reumatoide
negativo. El tratamiento específico inicial que se le debe indicar a este
paciente es con:

A Esteroides

B Analgésicos

Si usted no obtiene respuesta con el tratamiento establecido con su
paciente, el medicamento indicado es:

A tamoxifeno

B metotrexate

C celecoxib

D rituximab

Hombre de 65 años con diabetes mellitus desde hace 20 años. Es
encontrado en su domicilio con pérdida del estado de alerta. A la
exploración presenta ta 90/65, fc 11o/min, fr 21/min, glasgow 9.
Babinski dudoso, mucosa oral seca, pulsos disminuidos de intensidad.
Na 148 meq/l, k 3.5 meq/l, cl 110 meq/l, co2 18meq/l, glucosa 650
mg/dl, bun 45 mg/dl. El siguiente estado etiopatogénico explica el
cuadro clínico del paciente:

A cetoacidosis

B trombosis arteria cerebral media

C hipernatremia

Con base en su estado patogénico el tratamiento indicado sería:

A solución salina 0.9%

B terapia trombolítica

C solución salina 0.45%

D solución salina al 3%
A 24-year-old man comes to the office because of intermittent chest pain that began a few weeks
ago. You have been his physician for the past 2 years and he has been in otherwise good health.
He says that he is not having pain currently. A review of his medical record shows that his serum
cholesterol concentration was normal at a preemployment physical examination 1 year ago. You
have not seen him since that visit and he says that he has had no other complaints or problems
in the interim. He reminds you that he smokes a pack of cigarettes a day. When you question him
further, he says that he does not use any alcohol or illicit drugs. Although the details are vague,
he describes the chest pain as a substernal tightness that is not related to exertion.The finding on
physical examination that would be most consistent with costochondritis as the cause of his
chest pain is:
A crepitance over the second and third ribs anteriorly
B deep tenderness to hand pressure on the sternum
C localized point tenderness in the parasternal area
D normal physical examination
In light of the patient’s original denial of drug use, the most appropriate
next step to confirm a diagnosis of cocaine use is to:
A Ask the lab if serum is available for a toxicologic screening on his
previous blood sample.
B Call his family to obtain corroborative history.
C Obtain a plasma catecholamine concentration.
D Present your findings to the patient and confront him with the
suspected diagnosis.
Cocaine use is confirmed. The patient admits a possible temporal relationship
between his cocaine use and his chest pain. He expresses concern about
long-term health risks. He should be advised that:
A Cocaine-induced myocardial ischemia can be treated with beta-blockers.
B Death can occur from cocaine-induced myocardial infarction or
arrhythmia.
C The presence of neuropsychiatric sequelae from drug use indicates those
at risk for sudden death associated with cocaine use.
D Q wave myocardial infarction occurs only with smoked “crack” or
intravenous cocaine use.
A 45-year-old male comes to your office for his first annual checkup in the last 10 years. On first impression, he
appears overweight but is otherwise healthy and has no specific complaints. He has a brother with diabetes
and a sister with high blood pressure. Both of his parents are deceased and his father died of a stroke at age
73. He is a long-standing heavy smoker and only drinks alcohol on special occasions. On physical examination,
his blood pressure is 166/90 in the left arm and 164/88 in the right arm. The rest of the examination is
unremarkable. He is concerned about his health and does not want to end up on medication, like his siblings.
Regarding your initial recommendations, which of the following would be most appropriate?

A You should take no action and ask him to return to the clinic in 1 year for a repeat blood pressure check.
B You should immediately start him on an oral antihypertensive medication and ask him to return to the clinic
in 1 week.
C You should advise him to stop smoking, start a strict diet and exercise routine with the goal of losing weight,
and return to the clinic in 6 months.
D You should screen him for diabetes and evaluate him for other cardiovascular risk factors before proceeding
any further.
In the initial evaluation of a patient such as this, which of the following
should be routinely recommended?
A a urine microalbumin/creatinine ratio
B an echocardiogram
C thyroid function tests
D renal function tests (serum creatinine and blood urea nitrogen
[BUN])
Which of the following describes the patient’s blood pressure status?

A normal blood pressure


B prehypertension
C stage 1 hypertension
D stage 2 hypertension
Your patient returns to clinic a few weeks later for a follow-up appointment. Despite
having lost 3 lbs and increasing his activity to walking 2 mi three times per week, his
blood pressure remains elevated at 162/92. His initial evaluation revealed a fasting
blood sugar of 156 and a hemoglobin (Hgb) A1C of 7.5. Along with starting
hypoglycemic medications to control his diabetes, you recommend that he take an
antihypertensive medication. At this point, you decide to start the patient on
hypoglycemic medications to control his diabetes. What other intervention is
appropriate at this time?
A allow more time for the patient to practice lifestyle modifications
B start treatment with a two-drug combination
C start treatment with a thiazide diuretic only
D start treatment with a beta-blocker only
A 53-year-old female has made an appointment to see you concerning the recent onset of menopause. Her last
menstrual period was 8 months ago and, over the last year, she had noticed that her periods were becoming
lighter and less frequent. In addition, she has developed frequent hot flashes, and her mood has become
very labile. She wishes to know what your advice is regarding hormone replacement therapy (HRT). She has
heard recent reports in the news concerning an increased risk of developing cardiovascular complications,
especially heart attacks and strokes. Although she is in great health, her father died at age 50 of a massive
heart attack. Her mother is alive and well, and there is no history of breast cancer among the females in her
family.Regarding postmenopausal HRT, which of the following statements would be correct?
A Known benefits from HRT in postmenopausal women include a reduction in the incidence of osteoporosis
and bone fractures (particularly hip fractures).
B Known benefits from HRT in postmenopausal women include a cardioprotective effect, which reduces the
incidence of coronary artery disease (CAD) and myocardial infarction (MI).
C HRT increases the incidence of endometrial cancer in all patients
D Although HRT reduces vasomotor instability and hot flashes after menopause, this effect is short-lived and
there is no effect in mood stability.
Which of the following would be the strongest argument to avoid HRT
in this patient?
A HRT is unlikely to relieve her hot flashes.
B She has a positive family history of CAD.
C She is at high risk for developing breast cancer.
D She is at high risk for developing venous thromboembolism.
A 67-year-old male with a history of type II diabetes and hypertension is hospitalized with complaints of
retrosternal chest pain that radiates to the left arm and jaw. In the ED, an electrocardiogram (ECG) showed
S-T segment depressions in the inferior and lateral leads. He has been given the diagnosis of acute coronary
syndrome and admitted to the coronary care unit for further evaluation and treatment. Admission
laboratory values reveal a total cholesterol of 270, a lowdensity lipoprotein (LDL) of 190, and a high-density
lipoprotein (HDL) of 28. He is currently smoking a pack of cigarettes per day and lives a sedentary life. He is
clearly overweight and his blood pressure, despite medication, remains elevated at 150/88. His last HgbA1C
less than a month ago was 9.8%.After being discharged from the hospital, which of the following cholesterol
lowering regimens should be recommended to this patient?
A Low fat diet and exercise four times per week should reduce his cholesterol profile to acceptable levels.
B Starting a statin (3-hydroxy-3-methylglutaryl coenzyme A [HMG-CoA] reductase inhibitor) in addition to
smoking cessation, diet, and exercise may reduce his risk of developing further cardiovascular complications
C Starting niacin and recommending smoking cessation classes should be the first-line therapy in order to
increase his HDL and reduce his risk for further cardiovascular complications.
D There is no role for cholesterol-lowering medications in secondary prevention of cardiovascular disease.
In addition to diet, exercise, and smoking cessation, which of the
following would have the largest impact in reducing his cholesterol?
A controlling his blood pressure
B increasing his consumption of alcoholic beverages to three to four
glasses of wine per day
C adding thyroid hormone to his medications
D controlling his diabetes
A 48-year-old female with a history of mild congestive heart failure (CHF) treated with furosemide
presents to the emergency room (ER) for evaluation of 24 hours of epigastric pain, nausea, and
vomiting after eating a large meal in a restaurant. Previously, the patient had experienced
intermittent right upper quadrant pain after eating. On examination, the patient has a
temperature of 98.5°F and a pulse of 100. Her examination is remarkable for epigastric
tenderness to palpation, normal bowel sounds, and no rebound tenderness or guarding.
Laboratory studies are as follows:Leukocyte count 4800/mm3 Alanine aminotransferase (ALT)
258 U/L Aspartate aminotransferase (AST) 287 U/L Alkaline phosphatase, serum 350 U/L Bilirubin
(total) 2.0 mg/dL Bilirubin (indirect) 0.4 mg/dL Amylase 2865 U/L Lipase 3453 U/L Which of the
following is the most likely diagnosis?
A acute gastroenteritis
B acute gallstone pancreatitis
C drug-induced pancreatitis
D acute cholecystitis
What would the most appropriate next test to order be?

A abdominal x-ray
B abdominal computed tomographic (CT) scan
C abdominal ultrasound
D magnetic resonance imaging (MRI) of the abdomen
The patient is made NPO (nothing by mouth) and vigorously hydrated.
After 3 days, the amylase and lipase normalize, but the bilirubin rises
to 4.2 mg/dL. An endoscopic retrograde cholangiopancreatography
(ERCP) is performed, and the cholangiogram is obtained .What is the
best treatment option at this time?
A papillary dilation and stone extraction
B papillotomy (aka sphincterotomy) and stone extraction
C placement of a transpapillary stent in the biliary tree
D placement of a transpapillary stent in the pancreatic duct
A 35-year-old woman with a history of major depressive disorder is brought into the ED by her
boyfriend. He believes she may have overdosed on pain medication in an effort to hurt
herself. He gives you three medication bottles which he discovered empty and states that
they were nearly full before leaving for work earlier that morning. However, the timing of
the ingestion is unclear. All three of the bottles apparently held acetaminophen- containing
medications. Examination of the patient reveals a tired-appearing woman complaining of
nausea and right upper quadrant abdominal pain.Depletion of which of the following is
primarily responsible for the hepatotoxicity being experienced by the patient?
A N-acetyl-p-benzoquinone-imine (NAPQI)
B taurine
C citrulline
D glutathione
Which of the following pharmacologic treatments is most appropriate
at this time?
A N-acetylcysteine
B naloxone
C flumazenil
D physostigmine
Which of the following is the worst prognostic indicator if present in
this patient?
A arterial pH greater than 7.3
B arterial lactate greater than 3.5 mmol/L
C initiation of therapy 8 hours after acetaminophen ingestion
D “probable-risk” of hepatotoxicity by the Rumack-Matthew
nomogram
A 72-year-old male with type II diabetes, hypertension, and a history of
recurrent pneumonia is admitted to the Medical intensive care unit (ICU)
with a diagnosis of septic shock. His vital signs are: BP 80/60 mmHg, RR
24 breaths per minute, pulse 120 beats per minute (bpm), temp. 102.4°F,
O2 saturation 99% on room air. Of the choices listed below, what would
be your initial management?
A start IV dopamine
B start empiric IV broad-spectrum antibiotics
C bolus IV fluids
D intubate and start ventilator support
A42-year-old man without prior significant medical history comes to your office for evaluation of
chronic diarrhea of 12 months duration, although the patient states he has had loose stools for
many years. During this time he has lost 25 lbs. The diarrhea is large volume, occasionally
greasy, and nonbloody. In addition, the patient has mild abdominal pain for much of the day.
He has been smoking a pack of cigarettes a day for 20 years and drinks approximately five
beers per day. His physical examination reveals a thin male with temporal wasting and
generalized muscle loss. He has glossitis and angular cheilosis. He has excoriations on his
elbows and knees and scattered papulovesicular lesions in these regions as well.Which of the
following is the most likely diagnosis for this patient?
A chronic pancreatitis
B Crohn’s disease
C celiac sprue
D Whipple disease
Which of the following is the best test to confirm the suspected
diagnosis?
A abdominal CT scan with contrast
B small bowel x-ray
C esophagogastroduodenoscopy with small bowel biopsy
D colonoscopy with colonic biopsy
What is the most serious long-term complication this patient could
face?
A pancreatic cancer
B small bowel cancer
C gastric cancer
D colon cancer
A61-year-old man comes to your office for a checkup. He currently feels well and has no
focal complaints. He has a past medical history significant for wellcontrolled
hypertension, and his gallbladder was removed 3 years ago in the setting of acute
cholecystitis. He does not smoke and drinks one to two alcoholic beverages per day.
Family history is remarkable for colon cancer in his mother at age 45 and a brother at
age 49. He has a sister who developed endometrial cancer at age 53. He has never
undergone colon cancer screening and is interested in pursuing this. Which colorectal
cancer screening test would be best for this patient?
A virtual colonoscopy (aka CT colography)
B barium enema alone
C barium enema with flexible sigmoidoscopy
D colonoscopy
The patient’s family history is strongly suggestive of which of the
following?
A familial adenomatous polyposis (FAP) syndrome
B hereditary nonpolyposis colorectal cancer (HNPCC) syndrome
C Peutz-Jeghers syndrome
D Turcot syndrome
A 50-year-old female presents to your office for evaluation of solid food dysphagia without weight loss.
Symptoms have been present for 6 months and are progressive. The patient has had two episodes of
near impaction, but copious water ingestion and repeated swallows allowed the food bolus to pass. She
has never had to present to the ER for disimpaction. She drinks five to six beers per day, loves spicy
foods, and smokes a pack of cigarettes daily with a total lifetime history of 30 pack-years. She has had
intermittent heartburn symptoms for years and has not sought treatment. She takes hydrochlorothiazide
for hypertension. Review of symptoms reveals chronic cough. Physical examination is unremarkable.
Upper endoscopy reveals a distal esophageal stricture with inflammatory changes. Esophageal biopsies
reveal benign mucosa with chronic inflammation. Gastric biopsies are unremarkable. Helicobacter pylori
testing is negative.What is the most likely etiology of the patient’s stricture?
A alcohol ingestion
B tobacco use
C gastroesophageal reflux
D hydrochlorothiazide
What is the next best step in therapy for this patient?

A esophageal dilation
B histamine receptor antagonist therapy
C esophageal dilation with histamine receptor antagonist therapy
D esophageal dilation with PPI inhibitor therapy
The patient is at increased risk for which of the following illnesses?
A esophageal squamous cell cancer
B esophageal adenocarcinoma
C gastric cancer
D gastric lymphoma
A 65-year-old man presents to your office for evaluation of abdominal pain. The patient
states that he has epigastric pain that radiates to his back. The pain is worse with eating
and improves with fasting. The pain has been present for 6 months and is gradually
worsening. The patient has lost 15 lbs but feels his oral intake has been adequate. He
complains of greasy stools and frequent thirst and urination. Examination reveals a thin
male with temporal wasting and moderate abdominal pain with palpation. The patient
consumes approximately 10–15 beers per day and smokes a pack of cigarettes per day
for the past 20 years.What would be the best initial test to do in this patient?
A spot fecal fat collection
B 72-hour fecal fat collection
C CT scan of the abdomen
D ERCP
On further questioning, the patient reports that he recently had a
motor vehicle accident at night because he felt he could not see
clearly. The most likely cause of this symptom is which of the
following?
A vitamin B12 deficiency
B vitamin C deficiency
C vitamin D deficiency
D vitamin A deficiency
On further evaluation, the patient is found to be diabetic. He has an
elevated HgbA1C and fasting hyperglycemia. The patient is sent for
diabetic teaching sessions and begun on insulin therapy, but is unable to
achieve euglycemia. He experiences frequent bouts of symptomatic
hypoglycemia requiring ER visits. What is the most likely cause for these
episodes?
A insulin overdose
B impaired glucagon production
C inadequate oral intake
D vitamin K deficiency
The patient’s weight loss would be best treated by which of the
following regimens?
A pancreatic enzyme replacement therapy
B liquid caloric supplementation by mouth
C liquid caloric supplementation via gastrostomy tube
D total parenteral nutrition (TPN)
The patient’s abdominal pain worsens and his weight loss progresses
despite therapy, and you suspect that he may have a malignancy. If a
malignancy was present, which tumor marker would be most likely to
be elevated in this patient?
A carcinoembryonic antigen (CEA)
B cancer antigen (CA)-125
C α-Fetoprotein (AFP)
D CA-19-9
A 60-year-old woman arrives at your office for a routine physical examination. During the course
of her examination she asks you about osteoporosis. She is concerned about her risk for
osteoporosis, as her mother suffered from multiple vertebral compression fractures at the age
of 60. Your patient reports that she still smokes cigarettes (“although I know they are bad for
me”) and has one alcoholic beverage a week. She reports having had menopause 5 years ago
and experiencing a deep venous thrombosis approximately 20 years ago. She is proud of the
fact that she regularly exercises at the local fitness center. She has been taking 1500 mg of
calcium with 800 IU of vitamin D every day. You suspect that she is at risk for
osteoporosis.Which of the following tests is best to detect and monitor osteoporosis?
A plain film radiography
B dual photon absorptiometry
C single photon absorptiometry
D dual-energy x-ray absorptiometry (DEXA)
After performing the appropriate imaging study, you determine that
your patient has osteoporosis. Of the following choices, which is a
risk factor most likely contributing to her osteoporosis?
A active lifestyle
B late menopause
C cigarette smoking
D frequency of alcohol intake
After a thorough discussion with your patient, you determine that
pharmacologic intervention would be beneficial given the severity of
her osteoporosis. Which of the following is most appropriate for your
patient?
A estrogen replacement therapy
B combined HRT with estrogen and progestin
C alendronate
D calcitonin intranasal spray
A28-year-old male, well known to your clinic, presents for management of swelling, pain, and tenderness
that has developed in his left ankle and right knee. It has persisted for 1 month. Your patient reports that
he developed severe diarrhea after a picnic 1 month prior to the onset of his arthritis. During the interval
between the diarrhea and onset of arthritis, he developed a “pink eye” that lasted for 4 days. He denies
any symptoms of back pain or stiffness. You remember that he was treated with ceftriaxone and
doxycycline for gonorrhea 2 years ago, which he acquired from sexual activity with multiple partners.
Since that time, he has been in a monogamous relationship with his wife and has not had any
genitourinary symptoms. He promises that he has been faithful to his wife and has not engaged in
unprotected sexual activity outside his marriage. His physical examination is notable for a swollen left
ankle, swollen right knee, and the absence of penile discharge or any skin lesions.Which of the following
is the most likely diagnosis?
A pseudogout
B gout
C reactive arthritis
D resistant gonococcal arthritis
What would be the appropriate management for this patient’s
arthritis?
A Screen him for the suspected disease with HLA-B27 testing.
B Treat with daily indomethacin (150–200 mg daily).
C Start him on empiric antibiotics.
D Start treatment with prednisone 10 mg daily.
The patient’s symptoms do not respond to your initial therapeutic management.
You suspect that his condition is refractory to treatment. Which of the following
should you consider at this time?
A He may have human immunodeficiency virus (HIV) infection and should be
tested
B His condition will require high doses of prednisone (60 mg daily) for
adequate control.
C His joints are obviously not infected and should be directly injected with
corticosteroids.
D He must have a disseminated bacterial infection that will require IV
antibiotics
A 42-year-old man presents to your clinic with a 1-week history of pain and
inflammation involving his right first metatarsophalangeal (MTP) joint. He describes
the pain as sudden in onset and worse at night. He denies experiencing any fever or
traumatic injury to the joint and states that he has never had this type of pain before.
He denies any chronic medical conditions, any prior surgery, and any current
medication use. Besides an erythematous and exquisitely tender right first MTP joint,
the remainder of his physical examination is unremarkable.Aspiration of the patient’s
right first MTP joint space is likely to reveal which of the following?
A negatively birefringent crystals
B positively birefringent crystals
C nonbirefringent crystals
D acellular synovial fluid
Which of the following is true of the patient’s condition?
A It commonly presents in premenopausal women.
B It commonly presents as a monoarticular arthritis.
C Episodes of pain and inflammation become more frequent but
resolve more quickly as the disease progresses.
D The presence of tophi is a common early finding.
Which of the following interventions is most appropriate at this time
for your patient’s condition?
A probenecid
B allopurinol
C indomethacin
D sulfinpyrazone

También podría gustarte